Vous êtes sur la page 1sur 71

Copyright 2014 Delhi Academy of Medical Sciences, All Rights Reserved.

1/71

Test Information
Test Name

Pharmacology-SWT(MD/MS)

Total Questions

200

Test Type

Examination

Difficulty Level

Difficult

Total Marks

600

Duration

150minutes

Test Question Language:- ENGLISH


(1).

A drug being developed as an antiarrhythmic agent was studied in the laboratory using microelectrode techniques for measuring the
transmembrane potential. The results of this study are shown in Fig. Which of the following standard antiarrhythmic agents does the new
drug most resemble?

a. Adenosine
b. Ibutilide
c. Lidocaine
d. Procainamide
Solution. Ans-1: (b) Ibutilide
Ref: Read the text below
Sol :
The recorded transmembrane potential indicates that the new drug has no effect on the upstroke velocity but prolongs the action potential duration of the ventricular muscle
fiber.
This effect is characteristic of blocking potassium (IK) channels that open during repolarization, actions shown by group III antiarrhythmic drugs such as ibutilide. Adenosine
hyperpolarizes AV nodal cells and blocks propagation of impulses through this part of the heart.
It does not prolong the action potential. Lidocaine is a group IB drug and slows upstroke velocity of the action potential in susceptible cells but shortens, rather than prolongs,
the action potential duration.
Procainamide is a group IA drug that slows upstroke velocity as well as prolonging the action potential.

Correct Answer. b

(2).

Atropine inhibits all of the following, except


a. Sweating
b. Salivation
c. Gastric secretion
d. None
Solution. Ans-2: (d) None
Ref:Read the text below
Sol:
Atropine, an antimuscarinic, inhibits sweat and salivary secretion by M3 blockade and gastric secretion by M1 blockade.

Correct Answer. d

Copyright 2014 Delhi Academy of Medical Sciences, All Rights Reserved.

2/71

(3).

All of the following side effects are seen with ipratropium bromide, except
a. Urinary retention
b. Dryness of mouth
c. Scratching in trachea
d. Bad taste
Solution. Ans-3 : (a) Urinary retention
Ref:Read the text below
Sol:
Ipratropium bromide, given as inhalation, shows transient local side effects like dry mouth, scratching in trachea and bad taste.

But systemic effects like urinary retention are rare because of poor absorption from lungs and g.i.t.
Correct Answer. a

(4).

Which one of the following drugs increases gastro intestinal motility?


a. Glycopyrrolate
b. Atropine
c. Neostigmine
d. Fentanyl
Solution. Ans-4 : (c) Neostigmine
Ref:Read the text below
Sol:
Anticholinergics (e.g. Atropine, Glycopyrrolate) decreases gastro-intestinal motility while Cholinergic drugs (e.g. Cholinesterase inhibitors like Neostigmine) increase gastrointestinal motility.
Infact, Neostigmine is useful in post operative paralytic ileus.

Opioids (e.g. Fentanyl) decrease gastro-intestinal motility and are known to cause constipation
Correct Answer. c

(5).

Beta blockers that can be used in renal failure are all, except
a. Propranolol
b. Pindolol
c. Sotalol
d. Oxprenolol
Solution. Ans-5 : (c) Sotalol
Ref:Read the text below
Sol:
Unlike other beta-blockers, sotalol is excreted mostly unchanged in urine, and this takes place by glomerular filtration.

Since it is primarily excreted via urine, therefore it should not be used in renal failure.
About 40% of an oral dose of pindolol is excreted in urine as unchanged drug.
Propanolol and oxprenolol are mainly metabolized in liver and then excreted.
Correct Answer. c

Copyright 2014 Delhi Academy of Medical Sciences, All Rights Reserved.

3/71

(6).

A reasonable medication trial for complex partial seizures is


a. Haloperidol
b. Carbamazepine
c. Imipramine
d. Alprazolam
Solution. Ans-6: (b) Carbamazepine
Ref:Read the text below
Sol:
Carbamazepine is the preferred treatment for complex partial seizures.

The antipsychotic drug haloperidol and the antidpressant imipramine lower the seizure threshold.
The antipanic drug aprazolam is not indicated, although some other types of benzodiazepines are used as anttiepileptics.
Correct Answer. b

(7).

Heparin is contraindicated in all of the following conditions, except


a. SABE
b. Tuberculosis
c. Stable angina
d. Ocular or neurological surgery
Solution. Ans-7: (c) Stable angina
Ref:Read the text below
Sol:
Heparin is contraindicated in SABE, as embolism can occur. Also contraindicated in tuberculosis (hemoptysis) and ocular and neurosurgery.

Heparin in not contraindicated in stable angina,it is rather useful in unstable angina.


Correct Answer. c

(8).

Which of the following drugs does not cross the placenta


a. Carbimazole
b. Diazepam
c. Acenocoumarin
d. Heparin.
Solution. Ans-8: (d) Heparin.
Ref:Read the text below
Sol:
Heparin is a highly ionized large mucopolysacchride. So, it does not cross any biological membrane.

Therefore, it is not absorbed orally, dont cross blood brain barrier and dont cross placental barrier.
Correct Answer. d

Copyright 2014 Delhi Academy of Medical Sciences, All Rights Reserved.

4/71

(9).

A patient presents with an acute transmural myocardial infarction is made. All of the following statements regarding the use of a
thrombolytic agent in this patient are true, except
a. t-PA has a shorter half-life than streptokinase
b. T-PA administration is more likely to benefit this patients chances of survival than streptokinase
c. Streptokinase appears more effective when given less than 3 hours after the onset of symptoms
d. All thrombolytic agents can cause cardiac arrhythmias upon reperfusion.
Solution. Ans-9: (b) T-PA administration is more likely to benefit this patients chances of survival than streptokinase
Ref:Read the text below
Sol:
Both t-PA and streptokinase appear to reduce mortality equally when administered after the diagnosis of an acute transmural myocardial infarction.

However, streptokinase appears to be most effective when given less than 3 hours after the onset of symptoms.
All thrombolytic agents can cause cardiac arrhythmias upon reperfusion.
Correct Answer. b

(10).

The drugs such as procaine and pethidine are detoxified by which method
a. Conjugation
b. Hydrolysis
c. Oxidation
d. Reduction.
Solution. Ans-10 : (b) Hydrolysis
Ref:Read the text below
Sol:
Drugs which are esters (procaine), amides (lignocaine) and polypeptides (oxytocin) undergo hydrolysis in liver, intestine, plasma etc.

Correct Answer. b

(11).

A drug with first order kinetics after 4 half lives will be excreted by
a. 35%
b. 55%
c. 75%
d. 95%
Solution. Ans-11: (d) 95%
Ref:Read the text below
Sol:
A drug with first order kinetics will be excreted by 50% in 1 half-life, 75% in 2 half-lives, 88% in 3 half-lives and 94% in 4 half-lives.

Correct Answer. d

Copyright 2014 Delhi Academy of Medical Sciences, All Rights Reserved.

5/71

(12).

A highly ionized drug:


a. Is excreted mainly by the kidney
b. Can cross the placental barrier easily
c. Is well absorbed from the intestine
d. Accumulates in the cellular lipids.
Solution. Ans-12: (a) Is excreted mainly by the kidney
Ref:Read the text below
Sol:

1.
- A highly ionized drug can not cross a biological membrane. Therefore such a drug would not be reabsorbed through the renal tubular membrane and is excreted out by the
kidney.
- Actually, the very purpose of body metabolism is to change the lipophilic drugs into water soluble ionized drugs, so that they are excreted out of the body.
- As the ionized drug can not cross a membrane, it would not be absorbed well and is not able to cross placental barrier easily and is not able to enter inside the cells and also
is not lipophilic.
Correct Answer. a

(13).

Which of the following is an enzyme inhibitor?


a. Cimetidine
b. Phenobarbitone
c. Primidone
d. Carbamazepine.
Solution. Ans-13: (a) Cimetidine
Ref:Read the text below
Sol:
The important enzyme inhibitors are: Cimetidine, erythromycin, allopurinol, Ketoconazole, metronidazole and MAO-inhibitors.

Correct Answer. a

(14).

Gum hyperplasia, hirsutism and pseudolymphoma is caused by ingestion of which drug


a. Phenobarbitone
b. Phenytoin
c. Carbamazepine
d. Diazepam
Solution. Ans-14 : (b) Phenytoin
Ref:Read the text below
Sol:
Adverse effects of phenytoin are: Gum hypertrophy, hirsutism, hypersensitivity, megaloblastic anemia, osteomalacia, and rarely a lymphoma like syndrome.

Used during pregnancy, phenytoin may cause: Fetal hydantoin syndrome comprising of hypoplastic phalanges, cleft palate and microcephaly.
At high plasma levels, phenytoin may lead to: Cerebellar and ventricular manifestations ataxia, vertigo, diplopia, nystagmus, edema and gastric irritation.
Correct Answer. b

Copyright 2014 Delhi Academy of Medical Sciences, All Rights Reserved.

6/71

(15).

A 47-year-old man is under treatment for essential hypertension for 6 months. The patients hypertension is under control; however, he
now complains of impotence, vertigo, and difficulty in doing mental work. Laboratory results reveal a positive coombs test. Which
antihypertensive agent is capable of causing these effects?
a. Captopril
b. Hydralazine
c. Methyldopa
d. Prazosin
Solution. Ans-15 : (c) Methyldopa
Ref:Read the text below
Sol:
An antihypertensive agent capable of producing the adverse effects described in the question in methyldopa.

In addition, methyldopa can cause sedation, extraphyramidal signs, postural hypotension, and hepatic dysfunction; rebound hypertension can occur with sudden withdrawal of
methyldopa therapy.

Correct Answer. c

(16).

At the end of a balanced anesthesia technique with non-polarizing muscle relaxant, a patient recovered spontaneously from the effect of
muscle relaxant without any reversal. Which is the most probable relaxant the patient had received ?
a. Pancuronium
b. Gallamine
c. Atracurium
d. Vencuronium
Solution. Ans-16 : (c) Atracurium
Ref:Read the text below
Sol :
Atracurium, a competitive skeletal muscle relaxant, need no Neostigmine to reverse its effects.

This is because it undergoes self degradation (Hoffmans elimination).


Correct Answer. c

(17).

Pancuronium was used in a patient undergoing surgery as skeletal muscle relaxant, to achieve balanced anesthesia. Now, after the
surgery, which drug can be used to reverse the muscle relaxation ?
a. Physostigmine
b. Neostigmine
c. Acetylcholine
d. Adrenaline
Solution. Ans-17 : (b) Neostigmine
Ref:Read the text below
Sol :
Neostigmine is used to reverse the muscle relaxation caused by the competitive neuromuscular blockers (curare like drugs) e.g. pancuronium.

Neostigmine acts by cholinesterase inhibition as well as by direct action on the nicotinic receptors.
Correct Answer. b

Copyright 2014 Delhi Academy of Medical Sciences, All Rights Reserved.

7/71

(18).

Which of the following drug is contraindicated in Parkinsonism


a. Anticholinergic
b. Amoxapine
c. Carbidopa
d. Salicylates
Solution. Ans-18 : (b) Amoxapine
Ref:Read the text below
Sol :
Amoxapine is a unique drug as it is effective in both depression and psychosis.

As in Parkinsonism, dopamine is deficient and Amoxapine is antidopaminergic, hence contraindicated.


Correct Answer. b

(19).

In brain the breakdown of dopamine is inhibited by :


a. Deprenyl
b. Pyridoxine
c. Haloperidol
d. Bromocriptine
Solution. Ans-19: (a) Deprenyl
Ref:Read the text below
Sol :
Deprenyl is the trade name of Selegiline.

Selegiline inhibits MAO-A enzyme responsible for dopamine degradation.


So, Selegiline increases the dopamine availability in brain, hence its usefulness in Parkinsonism.
Correct Answer. a

(20).

Side effects of spironolactone are accelerated by the concurrent administration of


a. Thyroxine
b. ACE inhibitors
c. Salicylates
d. Paracetamol
Solution. Ans-20: (b) ACE inhibitors
Ref:Read the text below
Sol :
+
An important side effect of spironolactone is hyperkalemia (recall, it is a k sparing diuretic).
+
If ACE inhibitors (also cause K ) are concurrently administered, there may occur dangerous hyperkalemia and its consequences.

Correct Answer. b

Copyright 2014 Delhi Academy of Medical Sciences, All Rights Reserved.

8/71

(21).

Potassium supplements are given with


a. Spironolactone
b. Prednisolone
c. Thiazides
d. None of the above
Solution. Ans-21 : (c) Thiazides
Ref:Read the text below
Sol :
+
Thiazides causes significant potassium loss (hypokalemia), so K supplementation may be required.

On the contrary, spironolactone causes hyperkalemia.


Prednisolone, a glucocorticoid with negligible mineralocorticoid activity, doesnt cause hypokalemia.
Correct Answer. c

(22).

Half-life does not help in estimating


a. Margin of safety
b. Time taken to eliminate the drug completely from the body
c. Dosing schedule of drug
d. Concentration of drug in body at a particular time after its administration.
Solution. Ans-22: (a) Margin of safety
Ref:Read the text below
Sol:
Half-life is a pharmacokinetic parameter while margin of safety is a pharmacodynamic parameter.

Correct Answer. a

(23).

Drugs which induce cytochrome P-450 are all except :


a. Cimetidine
b. Barbiturates
c. Phenytoin
d. DDT
Solution. Ans-23: (a) Cimetidine
Ref:Read the text below
Sol:
Cimetidine and Ketoconazole are the enzyme inhibitors.

Following drugs are hepatic enzyme inducers Epilepsy drugs e.g. phenobaritone (Barbiturates), Phenytoin, Primidone, Carbamazepine,INH and Rifampicin (Anti-tubercular
drugs)

Correct Answer. a

Copyright 2014 Delhi Academy of Medical Sciences, All Rights Reserved.

9/71

(24).

Which of the following drugs cause IUGR


a. Methyldopa
b. Hydrallazine
c. Propranolol
d. Nifedipine
Solution. Ans-24 : (c) Propranolol
Ref:Read the text below
Sol:
Propranolol, a nonselective -blocker can cause intra-uterine growth retardation.

Correct Answer. c

(25).

Neostigmine does not cross blood brain barrier because of having


a. Secondary structure
b. Tertiary structure
c. Quaternary structure
d. Primary structure
Solution. Ans-25 : (c) Quaternary structure
Ref:Read the text below
Sol:
Unlike physostigmine, neostigmine has a quaternary nitrogen; hence, it is more polar and does not enter the CNS.

Its effect on skeletal muscle is greater than that of physostigmine, and it can stimulate contractility before it paralyzes. Neostigmine has moderate duration of action, usually
two to four hours

Correct Answer. c

(26).

The agent most useful for protection against the muscarinic manifestations of anti acetylcholinesterase poisoning is
a. Atropine
b. Di-isopropyl flourophosphate
c. Pralidoxime
d. Muscarine
Solution. Ans-26: (a) Atropine
Ref:Read the text below
Sol:
ANTI ACETYLCHOLINESTERASE POISONING
For Muscaranic effects- Atropine is drug of choice (Tachycardia is not a contraindication for atropine administration)

For Regeneration of AchE- Pralidoxime (2-PAM) Time dependent aging requires administration of PAM as soon as possible.
Correct Answer. a

Copyright 2014 Delhi Academy of Medical Sciences, All Rights Reserved.

10/71

(27).

Intrinsic renal failure is caused by nephrotoxicity of :


a. Aminoglycoside
b. Penicillin
c. Erythromycin
d. Ampicillin
Solution. Ans-27: (a) Aminoglycoside
Ref:Read the text below
Sol:
Intrinsic renal failure - Diseases of the renal parenchyma, specifically involving the renal tubules, glomeruli, interstitium

- ATN, ischemia, toxins (eg, aminoglycosides, radiocontrast, heme pigments, cisplatin, myeloma light chains, ethylene glycol)
- Interstitial diseases - Acute interstitial nephritis, drug reactions, autoimmune diseases (eg, systemic lupus erythematosus [SLE]), infiltrative disease (sarcoidosis, lymphoma),
infectious agents (Legionnaire disease, hantavirus)
- Acute glomerulonephritis

- Vascular diseases - Hypertensive crisis, polyarteritis nodosa, vasculitis


Correct Answer. a

(28).

Anticholinesterase drugs are ineffective against


a. Belladona Poisoning
b. Carbamate poisoning
c. Postoperative ileus
d. Cobra bite
Solution. Ans-28: (b) Carbamate poisoning
Ref:Read the text below
Sol:
Anticholinesterases have no role in the poisoning of Carbamates which are themselves anticholinesterase.
Uses of anticholinesterase drugs
Glaucoma : physostigmine

Myasthenia gravis : Neostigmine, Pyridostigmine.


Post-operative paralytic ileus : Neostigmine.
Post operative paralytic urinary retention : Neostigmine.
Post operative decurarization : Neostigmine,.
Cobra bite : Neostigmine along with Atropine.
Belladonna poisoning : Physostigmine
Overdosage of drugs having anticholinergic effects (e.g. Tricyclicantidepressants, Phenothiazines, antihistamincs : Physostigmine.
Alzheimers disease : Tacrine, Donepazil, Rivastigmine, Galantamine
Correct Answer. b

(29).

The following statements regarding benzodiazepines are true, except :


a. Binds to both GABAA and GABAB receptors
b. They have active metabolites
c. Decreases nocturnal gastric secretion in human being
d. Extensively metabolized by CYP enzymes
Solution. Ans-29: (a) Binds to both GABAA and GABAB receptors
Ref:Read the text below
Sol:
Benzodiazepines act on benzodiazepine receptors linked with GABAA receptors.

Please note that receptor acting drugs increasing GABAergic transmission are : - Benzodiazepines (act on BZD receptors linked with GABAA receptors) Baclofen (act on GABAB
receptors)

Correct Answer. a

Copyright 2014 Delhi Academy of Medical Sciences, All Rights Reserved.

11/71

(30).

An antipsychotic drug with antidepressant effect is


a. Clozapine
b. Flupenthixol
c. Pimozide
d. Penfluridol
Solution. Ans-30: (b) Flupenthixol
Ref:Read the text below
Sol:
Flupenthixol, an antipsychotic, in lower doses is also useful in depression. Similarly, amoxapine, an antidepressant, basically also has antipsychotic effects.

Correct Answer. b

(31).

Extrapyramidal symptoms are seen with


a. Reserpine
b. Phenothiazines
c. Clozapine
d. Risperidone
Solution. Ans-31: (b) Phenothiazines
Ref:Read the text below
Sol:
Phenothiazines major problem is extra-pyramidal side-effects, not found with atypical antipsychotics like clozapine and risperidone.

Reserpine, earlier a popular antipsychotic, fell into disrepute due to its tendency to cause severe depression and suicide.
Correct Answer. b

(32).

Which of the following drugs exerts its anti-seizure activity mainly as a result of interference with ion conductance through sodium
channels, which results in inhibition of high frequency repetitive firing of neurons?
a. Carbamazepine
b. Clonazepam
c. Ethosuximide
d. Gabapentin
Solution. Ans-32: (a) Carbamazepine
Ref:Read the text below
Sol:
Carbamazepine acts by inhibiting the sodium channels and act as the membrance stabilizer.

Therefore, it is not only a valuable antiepileptic but also very useful in mood stabilization and in the neuralgias.
Correct Answer. a

Copyright 2014 Delhi Academy of Medical Sciences, All Rights Reserved.

12/71

(33).

Physical withdrawal symptoms are seen with the following agents except
a. Pethidine
b. Opium
c. Alcohol
d. Cannabis
Solution. Ans-33: (d) Cannabis
Ref:Read the text below
Sol:
Withdrawal syndrome is not seen with cannabis, as there is no physical dependence seen with it.

Correct Answer. d

(34).

Undesirable side effects of morphine include


a. Nausea
b. Constipation
c. Dysphoria
d. All of the above
Solution. Ans-34 : (d) All of the above
Ref:Read the text below
Sol:
Morphine causes nausea, vomiting, constipation and sometimes dysphoria.

Correct Answer. d

(35).

All of the following statements about auranofin are true, except


a. It may be somewhat less effective than injectable gold for the treatment of rheumatoid arthritis
b. It is more likely to cause renal or mucocutaneous toxicity than injectable gold
c. A high incidence of diarrhea is associated with its use
d. Eosinophilia and proteinuria often occur with its use
Solution.
Ans-35: (b) It is more likely to cause renal or mucocutaneous toxicity than injectable gold
Ref:Read the text below
Sol:
Auranofin is a recently approved analong of gold thioglucose that can be administered orally.

Auranofin may be less effective than injectable gold for the treatment of arthritis.
It is less likely to cause renal and mucocutaneous toxicity.
There is a high incidence of diarrhea, but this is often self-limiting and rarely requires discontinuation of the therapy.
Cutaneous reactions, mucosal lesions, eosinophilia, and proteinuria are other common side effects.
Correct Answer. b

Copyright 2014 Delhi Academy of Medical Sciences, All Rights Reserved.

13/71

(36).

Aminophylline acts by
a. Stabilizing membrane of mast cells
b. Decreasing phoshodiesterase activity
c. Direct action on smooth muscles
d. Releasing catecholamines
Solution. Ans-36: (b) Decreasing phophodiesterase activity
Ref:Read the text below
Sol:
Aminophylline and other methylxanthines act bv
1. Inhibiting phosphodiesterase enzyme causing cAMP

2. Blocking adenosine receptors


2+
3. Releasing Ca from sarcoplasmic reticulum in cardiac and skeletal muscles.
Correct Answer. b

(37).

Bisacodyl acts as a purgative due to its


a. Irritant action
b. Bulk formation
c. Osmotic action
d. Emollient effect
Solution. Ans-37: (a) Irritant action
Ref:Read the text below
Sol:
Bisacodyl is a commonly used purgative.

It acts by irritating gastric mucosa and myenteric plexus.


Correct Answer. a

(38).

Erythromycin is the drug of choice for all except


a. Diphtheria
b. Whooping cough
c. Pneumocystis carinii
d. Chlamydia trachomatis
Solution. Ans-38: (c) Pneumocystis carinii
Ref:Read the text below
Sol:
Erythromycin is the drug of choice in diphtheria, pertussis (whooping cough) and Chlamydia infections.

It is not useful in pneumocystis carinii pneumonia (PCP),in PCP co-trimoxazole is the drug of choice and pentamidine is the alternative.
Correct Answer. c

Copyright 2014 Delhi Academy of Medical Sciences, All Rights Reserved.

14/71

(39).

The adverse effects of amphotericin-B are all except


a. Gastrointestinal upset
b. Thrombophlebitis
c. Renal damage
d. Cholestatic jaundice
Solution. Ans-39 : (d) Cholestatic jaundice
Ref:Read the text below
Sol:
Fever with chills and aches, nausea and vomiting are common with amphotericin, on every i.v. injection (however tolerance occurs).

Paracetamol or steroid treatment reduces this.


Thrombophlebitis of the injected vein may occur.
Nephrotoxicity is the most important problem with amphotericin-B causing azotemia, acidosis and inability to concentrate urine. Cholestatic jaundice is not reported till date.
Correct Answer. d

(40).

Which of these is not caused by Amphotericin-B


a. Azotemia
b. Glomerulonephritis
c. Hypokalemia
d. Renal tubular acidosis
Solution. Ans-40 : (b) Glomerulonephritis
Ref:Read the text below
Sol:
Amphotericin-B actually causes renal tubular damage and not glomerulonephritis.

This nephrotoxicity is the most important long term toxicity of amphotericin-B.


It leads to inability to concentrate urine, azotemia, acidosis, hypokalemia and reduced glomerular filtration.
The nephrotoxicity caused by amphotericin-B reverses slowly and incompletely after stoppage of the therapy.
Correct Answer. b

(41).

Which of the following anti-metabolite acts as an antifungal agent :


a. Paclitaxel
b. 5-Flucytosine
c. Chlorambucil
d. Decarbazine
Solution. Ans-41: (b) 5-Flucytosine
Ref:Read the text below
Sol:
5-Flucytosine is the only anti-metabolite (anticancer) drug used as an antifungal agent.

5-Flucytosine is taken up by the fungal cells and convert it into 5-Fluorouracil.


It is then incorporated into its RNA and then inhibits the protein synthesis and growth of fungi.
5-Flucytosine is used most commonly with Amphotericin-B for systemic fungal infections caused by Cryptococcus and Candida.
Correct Answer. b

Copyright 2014 Delhi Academy of Medical Sciences, All Rights Reserved.

15/71

(42).

True about ipratropium bromide are all, except


a. Used by inhalation
b. Increases intra-ocular pressure usually
c. Causes dryness of mouth
d. Causes scratching in trachea.
Solution. Ans-42: (b) Increases intra-ocular pressure usually
Ref:Read the text below
Sol :
Ipratropium bromide, given as inhalation, can cause dryness of mouth, bad taste, scratching in trachea and cough.

But systemic effects like increase in intra-ocular pressure are rare because of poor absorption from lungs and GIT.
Correct Answer. b

(43).

Botulinum toxin blocks neuromuscular transmission by which of the following mechanism :


a. Closure of Ca++ channels at the presynaptic membrane
b. Closure of Na+ channels at the postsynaptic membrane
c. Opening of K+ channels at the presynaptic membrane
d. Opening of Cl channels at the postsynaptic membrane
Solution. Ans-43 : (a) Closure of Ca++ channels at the presynaptic membrane
Ref: Read the text below
Sol :
++
Botulinum toxin blocks neuromuscular transmission by closure of Ca
channels at the presynaptic membrane.
Botulinum Toxin

Botulinum toxin causes flaccid paralysis by blocking the release of acetylcholine at the neuromuscular junction.
2+
Botulinum toxin blocks neuromuscular transmission by blocking presynaptic Ca channels.

Correct Answer. a

(44).

One of the following activities is not mediated through 2 adrenergic receptors :


a. Stimulation of lipolysis
b. Increased hepatic gluconeogenesis
c. Increased muscle glycogenolysis
d. Smooth muscle relaxation
Solution. Ans-44: (a) Stimulation of lipolysis
Ref: Read the text below
Sol :
Stimulation of lipolysis is mediated through 3 not the 2 adrenergic receptors.

Fat-lipolysis (3) increased blood FFA, calorigenesis


Correct Answer. a

Copyright 2014 Delhi Academy of Medical Sciences, All Rights Reserved.

16/71

(45).

Which of the following is a selective estrogen receptor modulator ?


a. Raloxifene
b. Mifepristone
c. Danazol
d. Anastozole
Solution. Ans-45 : (a) Raloxifene
Ref: Read the text below
Sol :
Raloxifene
Recently introduced SERM

An estrogen partial agonist in bone and cardiovascular system, but an antagonist in endometrium and breast.
High affinity for both ER and ER and has a distinct DNA target the Raloxifene response element.
Absorbed orally but has low bio-availability due to extensive first pass metabolism
Major route of excretion is feces.
Correct Answer. a

(46).

Which of the following is true statement regarding Metoclopramide :


a. Inhibits cholinergic smooth muscle stimulation in the gastrointestinal tract
b. Decreases lower esophageal sphincter pressure
c. It increases intestinal peristalsis
d. Enhances colonic motility
Solution. Ans-46 : (c ) It increases intestinal peristalsis
Ref: Read the text below
Sol :
Metoclopramide doesnt enhance colonic motility.

It increases intestinal peristalsis to some extent, but has no significant action on colonic motility and gastric secretion.
Correct Answer. c

(47).

Blockade of nerve conduction by a local anesthetic is characterized by


a. Greater potential to block a resting nerve as compared to a stimulated nerve
b. Need to cross the cell membrane to produce the block
c. Large myelinated fibres are blocked before the unmyelinated fibers
d. Cause consistent change of resting membrane potential.
Solution. Ans-47: (b) Need to cross the cell membrane to produce the block
Ref: Read the text below
Sol :
Blockade of nerve conduction by a local anesthetic is characterized by need to cross the cell membrane to produce the block.

Correct Answer. b

Copyright 2014 Delhi Academy of Medical Sciences, All Rights Reserved.

17/71

(48).

Correct about the use of lithium is an important component of management of manic-depressive psychosis :
a. Is associated with delayed (<2 weeks) electrolyte disturbances
b. Can be given along for acute episodes
c. Monitoring of serum lithium concentration is seldom useful for guiding dose adjustment
d. Cause benign and reversible depression of T wave on ECG
Solution. Ans-48 : (d) Cause benign and reversible depression of T wave on ECG
Ref: Read the text below
Sol :
Correct about the use of lithium is an important component of management of manic depressive psychosis, cause benign and reversible depression of T wave on ECG.

Lithium causes T wave depression (effects are similar to hypokalemia).


Correct Answer. d

(49).

Prostaglandin analogs have therapeutic utility in the following, except :


a. Palliative treatment of patient ductus arteriosus
b. Pulmonary hypertension
c. Impotence
d. Inflammatory bowel disease
Solution. Ans-49 : (d) Inflammatory bowel disease
Ref: Read the text below
Sol :
Prostaglandin analogs havetherapeutic utility in the following, except inflammatory bowel disease.

Correct Answer. d

(50).

As compared to unfractionated heparin, low molecular weight heparins :


a. Are absorbed more uniformly when given subcutaneously
b. Require more frequent lab monitoring
c. Can be given to patient with heparin induced thrombocytopenia
d. A higher risk of osteopenia
Solution. Ans-50 : (a) Are absorbed more uniformly when given subcutaneously
Ref: Read the text below
Sol :
Low molecular weight (LMW) Heparins
Different articoagulation profile.

Selectively inhibit factor Xa with little effect on IIa


Act only by inducing conformational change in AT-III and not by bringing together ATIII and thrombin.
Lesser antiplatelet action and less interference with bleeding.
Indications of LMW heparins :
1. Prophylaxis of DVT and pulmonary embolism in high risk patients undergoing surgery, stroke or other immobilized patients.

2. Treatment of established DVT


3. Unstable angina.
4. To maintain patency of cannulae and shunts in dialysis patients and in extracorporeal circulation
Correct Answer. a

Copyright 2014 Delhi Academy of Medical Sciences, All Rights Reserved.

18/71

(51).

The time required to achieve maximal concentration (Tmax) :


a. Is important for designing multiple dosage regimen
b. Is the time when rate of absorption exceeds rate of elimination
c. Depends on the type of formulation in case of oral medications
d. Is a critical parameter for acute conditions
Solution. Ans-51 : (c) Depends on the type of formulation in case of oral medications
Ref: Read the text below
Sol :
The time required to achieve maximal concentration (Tmax), Depends on the type of formulation in case of oral medications.

Correct Answer. c

(52).

For oral iron supplements used for iron deficiency anemia :


a. Tolerable dose will deliver 40 to 60 mg of iron per day
b. Mass of total salt is important in determining daily dose
c. Treatment should be stopped as soon as normal hemoglobin level is reached
d. Desired rate of hemoglobin improvement is 0.5 mg per day
Solution. Ans-52 : (a) Tolerable dose will eliver 40 to 60 mg of iron per day
Ref: Read the text below
Sol :
Treatment of iron deficiency anemia
The elemental iron content, not the quantity of iron compounds per dose unit should be taken into consideration.

A rise in Hb level 0.5-1 g / dl per week is an optimum response to iron therapy.


Therapy should be continued till normal normal Hb level is attained and 2-3 months thereafter to replenish the stores, because after correction of anemia, iron absorption is
slow.
In an iron deficient individual, about 50-100 mg of iron can be incorporated into hemoglobin daily, and about 25% of oral iron given as ferrous salt can be absorbed.

A total dose of 200 mg elemental iron given daily in 3 divided doses produces the maximal haemopoetic response.
Correct Answer. a

(53).

Which diuretic could be considered appropriated for combining with ACE inhibitors ?
a. Spironolactone
b. Eplerenone
c. Hydrochlorothiazide
d. Amiloride
Solution. Ans-53 : (c) Hydrochlorothiazide
Ref: Read the text below
Sol :
ACE inhibitors, when used alone control hypertension in 50% patients, and addition of a diuretic/ blocker extends efficacy to 99%, because of super additive synergism, only a
low dose of diuretic (12.5 mg of hydrochlorthiazide, rarely 25 mg) needs to be added.

Correct Answer. c

Copyright 2014 Delhi Academy of Medical Sciences, All Rights Reserved.

19/71

(54).

Which of the following drugs is effective against Pseudomonas infection ?


a. Ampicillin
b. Ceftriaxone
c. Colistin
d. Cefixime
Solution. Ans-54: (c) Colistin
Ref: Read the text below
Sol :
Colistin is effective against Pseudomonas infection.

Polypeptide like Polymyxin B and Colistin may be used.


Colistin is more potent in Pseudomonas, Salmonella, Shigella.
Correct Answer. c

(55).

Ethambutol causes :
a. Retrobulbar neuritis
b. Deafness
c. Red urine
d. All of the above
Solution. Ans-55 : (a) Retrobulbar neuritis
Ref: Read the text below
Sol :
Ethambutol (commonly abbreviated EMB or simply E) is a bacteriostaticantimycobacterialdrug prescribed to treat tuberculosis.
It is usually given in combination with other tuberculosis drugs, such as isoniazid, rifampicin and pyrazinamide.
It is sold under the trade names Myambutol and Servambutol.
Adverse effects
[2]
Optic neuritis. Hence contraindicated in children below 6 yrs of age.

Red-green color blindness


Peripheral neuropathy
Arthralgia
Hyperuricaemia
Vertical nystagmus
Milk skin reaction

Correct Answer. a

(56).

Pyridoxine deficiency can cause all, except :


a. Convulsion in infant
b. Anemia
c. Peripheral neuritis
d. Growth retardation
Solution.
Ans-56 : (d) Growth retardation
Ref: Read the text below
Sol :
Pyridoxine deficiency
Symptoms include epithelial changes

Severe deficiency can lead to peripheral neuropathy, abnormal EEG, personality changes including depression and confusion.
In infants, diarrhea, seizures and anemia has been reported.
Microcytic, hypochromic anemia is due to diminished hemoglobin synthesis, as this is required in Hb synthesis.
Correct Answer. d

Copyright 2014 Delhi Academy of Medical Sciences, All Rights Reserved.

20/71

(57).

All of the following statements are true about theophylline, except


a. Increase in dose required in cardiopulmonary diseases
b. Increases cAMP
c. Increase in dose required in smokers
d. Inhibits phosphodiesterase
Solution. Ans-57 : (a) Increase in dose required in cardiopulmonary diseases
Ref:Read the text below
Sol :
Theophylline (including other methyl-xanthines) act by inhibiting phosphodiesterase enzyme, thus increasing cAMP.

Smoking induces liver enzymes causing increased metabolism of theophylline, thus require a higher dose.
Factors which need dose reduction are : Age>60 years, pneumonia and drugs which inhibit theophyslline metabolism.
Cardiopulmonary diseases may demand reduction and not an increase in dosage.
Correct Answer. a

(58).

All of the following agents are used locally to control bleeding, except
a. Thromboplastin
b. Thrombin
c. Fibrinogen
d. Gelatin foam
Solution. Ans-58 : (c) Fibrinogen
Ref:Read the text below
Sol :
The agents used locally to stop bleeding are called local hemostatics or stypitcs.

These agents are thrombin, fibrin sheets/foam, thromboplastin (as in russel viper venom), gelatin foam soaked in saline/thrombin, vasoconstrictors (e.g. adrenaline) and
astringents (e.g. tannic acid).
Fibrinogen, however, is not used locally as a styptic.

It issued systemically, as I.V. infusion to control bleeding, as in hemophilia.


Correct Answer. c

(59).

The oral anticoagulant levels are increased by following, except


a. Phenytoin
b. Quinidine
c. Quinine
d. Griseofulvin
Solution. Ans-59 : (d) Griseofulvin
Ref:Read the text below
Sol :
Phenytoin, quinidine and quinine increase plasma levels of the oral anticoagulants.

However, griseofulvin, an enzyme inducer, decreases the plasma levels of the oral anticoagulants by increasing their metabolism.
Correct Answer. d

Copyright 2014 Delhi Academy of Medical Sciences, All Rights Reserved.

21/71

(60).

The following are contraindications to thrombolytic therapy, except


a. Hypertension (diastolic B.P. 90 mm. Hg)
b. Serious G.I. bleeding within 3 months
c. Acute pericarditis
d. Aortic dissection
Solution. -NACorrect Answer. a

(61).

All of the following statements about ticlopidine are true, except


a. Directly interacts with platelet membrane, gpIIb/IIIa receptiors
b. Onset of action is delayed
c. Duration of action is long
d. It is used as an alternative to aspirin in patients with cardiovascular disease.
Solution. Ans-61 : (a) Directly interacts with platelet membrane, gpIIb/IIIa receptiors
Ref:Read the text below
Sol :
Ticlopidine is not a glycoprotein IIb/IIIa inhibitor.

Glycoprotein IIb/IIIa receptor inhibitiors are new antiplatelet agents e.g., abciximab, eptifibatide and tirofiban.
Ticlopidine blocks P2Y receptors on platelets and inhibits ADP mediated platelet aggregation.
It acts slowly and has a long duration of action.
It is a useful alternative to aspirin, as antiplatelet drug.
Correct Answer. a

(62).

Mechanism of action of heparin is


a. Increased interaction between thrombin and antithrombin
b. Decreased synthesis of vitamin K dependent clotting factors
c. Degradation of clotting factors
d. Decreased synthesis of fibrinogen.
Solution. Ans-62: (a) Increased interaction between thrombin and antithrombin
Ref: Read the text below
Sol :
Heparin activates antithrombin III and increases its interaction with thrombin and other factors.

Therefore, conversion of fibrinogen to fibrin is prevented, causing instant anticoagulation.


Oral anticoagulant (e.g. warfarin) act by inhibiting vitamin K dependent synthesis of clotting factors in liver.
Correct Answer. a

Copyright 2014 Delhi Academy of Medical Sciences, All Rights Reserved.

22/71

(63).

Which of the following drug does not cross the placenta


a. Heparin
b. Phenytoin
c. Diazepam
d. Lithium
Solution. Ans-63: (a) Heparin
Ref:Read the text below
Sol :
Heparin, being a highly ionized molecule, does not cross the placenta.

Heparin, therefore, is absolutely safe in pregnancy.


Correct Answer. a

(64).

Drugs causing hypoglycemia is:


a. Diazoxide
b. Theophylline
c. 10% glucose
d. Quinine
Solution. Ans-64 : (d) Quinine
Ref:Read the text below
Sol :
Drugs casuing Hypoglycemia :
-Insulin
-Pentamidine
-Sulfonamides

-Sulfonylureas
-Ethanol
-ACE inhibitors

Correct Answer. d

(65).

Which one of the following statements about biguanides is not true :


a. Dont stimulate insulin release
b. Decrease hepatic glucose production
c. Renal Dysfunction is not a contraindication for their use
d. Can be combined with sulfonylureas.
Solution. Ans-65 : (c) Renal Dysfunction is not a contraindication for their use
Ref:Read the text below
Sol :
BIGUANIDES
Dont stimulate insulin release. Main action is to decrease glucose output from liver by suppressing gluconeogenesis.

Use as monotherapy in obese diabetics. Can be combined usefully with sulfonylureas in uncontrolled diabetes.
Not metabolized and excreted unchanged by the kidneys; contraindicated in renal insufficiency.
Major adverse effect is lactic acidosis.
At normal therapeutic dose, dont cause hypoglycemia.
Correct Answer. c

Copyright 2014 Delhi Academy of Medical Sciences, All Rights Reserved.

23/71

(66).

New drug for increasing sensitivity of cells to insulin is :


a. Glibenclamide
b. Venlafaxine
c. Proglitazone
d. Glipizide
Solution. Ans-66 : (c) Proglitazone
Ref:Read the text below
Sol :
A new class of anti-diabetic drugs has been introduced in the market, the Thiazolidinediones.

Their main action is to increase the sensitivity of cells to insulin.


Thiazolidinediones :
Cliglitazone

Troglitazone
Pioglitazone
Rosiglitazone.
These novel drugs increase expression of insulin responsive genes. They can even reverse the insulin resistance.
Correct Answer. c

(67).

Which of the following drugs in not penicillinase resistant


a. mpicillin
b. Oxacillin
c. Methicillin
d. Nafcillin
Solution. Ans-67 : (a) Ampicillin
Ref:Read the text below
Sol :
Ampicilin in degraded by penicillinase enzyme produced by some bacteria, thus causing resistance.

Penicillinase resistant penicillins are : Methicillin, nafcillin, oxacillin, cloxacillin, dicloxacillin and flucloxacillin.
Correct Answer. a

(68).

The drug of choice in actinomyces Israeli infection is


a. Penicillin G
b. Streptomycin
c. Miconazole
d. Amphotericin B
Solution. Ans-68 : (a) Penicillin
Ref:Read the text below
Sol :
Actinomyces Israeli causes actinomycosis which may envolve cervicofacial, thoracic, abdominal and other regions.

Penicillin-G is the first drug of choice in actinomycosis.


Ampicillin doxycycline and erythromycin are the alternative drugs in this condition.
Correct Answer. a

Copyright 2014 Delhi Academy of Medical Sciences, All Rights Reserved.

24/71

(69).

The drug of choice in the treatment of gonococcal infection is


a. Gentamicin
b. Co-trimoxazole
c. Carbenicillin
d. Ampicillin
Solution. Ans-69 : (d) Ampicillin
Ref:Read the text below
Sol :
Ampicillin or amoxicillin or procaine penicillin is the drug of choice against gonococcus not producing penicillinase.

As nowadays, many strains have become resistant, the drugs of choice have been changed in gonorrhea.
The first choice is either a cephalosporin (e.g. ceftriaxone) or a quinolone (e.g. ciprofloxacin).
Second choice is doxycycline and the third choice being spectinomycin.
Correct Answer. d

(70).

Drugs inhibiting IL-2:


a. Cycloserine
b. Cyclosporine
c. OKT-3
d. None
Solution. Ans-70: (b) Cyclosporine
Ref:Read the text below
Sol:
Cyclosporine and Tacrolimus specifically inhibit antigen stimulated activation & proliferation of helper T-cells as well as expression of various cytokines including IL-2

Correct Answer. b

(71).

Etanercept acts by one of the following mechanisms:


a. By blocking tumor necrosis factor
b. By blocking bradykinin synthesis
c. By inhibiting cyclo-oxygenase-2
d. By blocking lipoxygenase
Solution. Ans-71: (a) By blocking tumor necrosis factor
Ref:Read the text below
Sol:
Etanercept (trade name Enbrel) is a drug that treats autoimmune diseases by interfering with the tumor necrosis factor (TNF, a part of the immune system) by acting as a TNF
inhibitor.
Etanercept is a fusion protein produced through expression of recombinant DNA.

That is, it is a product of a DNA "construct" engineered to link the human gene for soluble TNF receptor 2 to the gene for the Fc component of human immunoglobulin G1 (IgG1).
Correct Answer. a

Copyright 2014 Delhi Academy of Medical Sciences, All Rights Reserved.

25/71

(72).

Frusemide differs from thiazides in respect of


a. Action on proximal and distal tubule
b. Onset and duration of action
c. Potassium loss
d. Potentiate ADH action
Solution. Ans-72: (b) Onset and duration of action
Ref:Read the text below
Sol:
Onset of action of frusemide (=furosemide) is less than 45 min, while that of thiazides is more than 45 min.

Duration of action of frusemide is les than 6 hrs, while that of thiazides is more than 6 hrs.
Frusemide acts on thick ascending limb of loop of Henle and does not potentiate ADH.
Both of them cause potassium loss.
Correct Answer. b

(73).

Which of the following statements is not true about diuretics


a. Acetazolamide is a carbonic acid anhydrase stimulant
b. Thiazides act on cortical diluting segment or the early distal tubule
c. Frusemide is a high ceiling diuretic
d. Spironolactone is an aldosterone antagonist
Solution. Ans-73: (a) Acetazolamide is a carbonic acid anhydrase stimulant
Ref:Read the text below
Sol:
Acetazolamide is a carbonic anhydrase inhibitor. It is a weak diuretic, rather used to alkalinize the urine.

Also, acetazolamide is used as an adjuvant drug in glaucoma and epilepsy.


May be useful in acute mountain sickness and periodic paralysis also.
Correct Answer. a

(74).

Cough is an adverse effect seen with the intake of


a. Thiazides
b. Nifedipine
c. Enalapril
d. Prazosin
Solution. Ans-74 : (c) Enalapril
Ref:Read the text below
Sol:
Dry cough is a typical side effect seen with ACE inhibitors (e.g. enalapril etc.).

Dry cough is due to increased inflammatory substances like bradykinin and substance P whose degrading enzyme (kininase) is also inhibited by ACE inhibitors.
Correct Answer. c

Copyright 2014 Delhi Academy of Medical Sciences, All Rights Reserved.

26/71

(75).

All of the following antihypertensives decrease plasma renin activity, except


a. Clonidine
b. Methyldopa
c. Atenolol
d. Chlorthiazide
Solution. Ans-75 : (d) Chlorthiazide
Ref:Read the text below
Sol:
Clonidine and methyldopa are central sympatholytics. Atenolol also decreases beta adrenergic pathway.

So, the above drugs decrease renin release.


Diuretics like chlorthiazide increase renin release.
Correct Answer. d

(76).

The role of anticoagulation with warfarin has been established for


a. Immediate therapy of cerebral thrombosis
b. Chronic therapy of venous thrombosis
c. Prevention of heart attack
d. Prinzmetals or variant angina
Solution. Ans-76 : (b) Chronic therapy of venous thrombosis
Ref:Read the text below
Sol:
Anticoagulant therapy is of proven efficacy in the prevention and treatment of venous thrombosis and pulmonary embolism. While anticoagulants are of little value in cerebral
thrombosis and prevention of heart attacks (aspirin preferred).
In case of Prinzmetals/variant angina to prevent Ac. M.I. the treatment is aspirin + heparin following by warfarin.

Correct Answer. b

(77).

A 33-year-old with a history of asthma is being treated for symptoms of hypertension. Which of the following beta-blockers would be an
appropriate therapy for this patient?
a. Isoproterenol
b. Labetalol
c. Metoprolol
d. Propranolol
Solution. Ans-77: (c) Metoprolol
Ref:Read the text below
Sol:
If the patient has asthma, you should select a drug that blocks beta1 receptors without affecting the beta2-receptors without affecting the beta2-receptors found in the
respiratory smooth muscle.
Therefore, a selective beta1-blocker (such as metoprolol) would be appropriate.

Correct Answer. c

Copyright 2014 Delhi Academy of Medical Sciences, All Rights Reserved.

27/71

(78).

Which of the following regarding Infliximab is most true


a. Is a monoclonal antibody to the glycoprotein IIb-IIIa receptor
b. Is authorised for the treatment of severe ulcerative colitis
c. Is licensed for the treatment of rheumatoid arthritis
d. It prevents relapse of Crohn's disease in patients who are in remission
Solution. Ans 78 : (c) Is licensed for the treatment of rheumatoid arthritis
Reference Read the text below
Sol:
Infliximab is a monoclonal antibody to TNF alpha. It is licensed for the treatment of:- (1). Severe active Crohn's disease refractory to corticosteroid or immunosuppressant
therapy and for refractory fistulas.
Rheumatoid arthritis in adults whose response to disease-modifying antirheumatic drugs has been inadequate. Before starting therapy and throughout treatment, patients
should be evaluated carefully for tuberculosis as there have been reports of the onset or reactivation of TB including miliary TB and some unusual extrapulmonary TB.
Infliximab must be given concomitantly with methotrexate and requires IV infusion in a hospital setting. Some other monoclonal antibodies in clinical use include
(1)Digibind - digoxin-binding antibody for treatment of overdoses (increases clearance); (2)Abciximab: glycoprotein IIbIIIa receptor (for unstable angina);
(3)Pexelizumab: anti-C5 (complement) - anti-inflammatory: reduces MI and death following CABG.)

Correct Answer. c

(79).

A 17 year old female is admitted with an oculogyric crisis. Which of the following statements concerning this case is correct?
a. She should be observed without treatment
b. She is likely to have been prescribed Olanzapine
c. She should be treated with parenteral procyclidine
d. She should receive procyclidine as long-term prophylaxis
Solution. Ans 79: (c) She should be treated with parenteral procyclidine
Reference Read the text below
Sol:
Oculogyric crisis is an acute dystonic reaction of the face/eyes and is usually a consequence of typical neuroleptic drugs such as Haloperidol,Chlorpromazine but is unusual
with newer agents such as Olanzapine, Clozapine.
The condition is often precipitated by re-introduction of the agent.

The condition should be treated with procyclidine (usually IV or IM) or Benztropine. Chronic treatment beyond a couple of days is not required.
Correct Answer. c

(80).

Long acting beta 2 agonist


a. Orciprenaline
b. Salmeterol
c. Albuterol
d. Pibuterol
Solution. Ans-80: (b) Salmeterol
Ref:Read the text below
Sol:
Salmeterol is the first long acting beta 2 stimulant, with slow onset of action, used by inhalation on a twice schedule on a maintanance therapy, in nocturnal asthma but not for
acute symptoms.
In COPD long acting beta 2 agonists are superior to short acting agonists and equivalent to anti cholinergics.

Orciprenaline is a beta adrenergic stimulant acting on both beta 1 and beta 2, more specific for beta 2. It is used in bronchial asthma (beta 2 receptors), sick sinus syndrome
(beta 1 receptors). Its side effects are tachycardia (beta 1), skeletal muscle tremor (beta 2).

Correct Answer. b

Copyright 2014 Delhi Academy of Medical Sciences, All Rights Reserved.

28/71

(81).

All of following are contraindications for use of lidocaine except


a. Pacemaker
b. SA node block
c. Postherpetic neuralgia
d. Use of amiodarone
Solution. Ans-81: (c) Postherpetic neuralgia
Ref:Read the text below
Sol:
CONTRAINDICATIONS FOR THE USE OF LIDOCAINE INCLUDE:
Heart block, second or third degree (without pacemaker)

Severe sinoatrial block (without pacemaker)


Serious adverse drug reaction to lidocaine or amide local anaesthetics
Concurrent treatment with quinidine, flecainide, disopyramide, procainamide (Class I antiarrhythmic agents)
Prior use of Amiodarone hydrochloride
Hypotension not due to Arrhythmia
Bradycardia
Accelerated idioventricular rhythm
Pacemaker
Correct Answer. c

(82).

The following drug is not useful for MRSA


a. Cefaclor
b. Cotrimoxazole
c. Ciprofloxacin
d. Vancomycin
Solution. Ans-82: (a) Cefaclor
Ref:Read the text below
Sol:
Drugs of first choice for MRSA Vancomycin + gentamicin + rifampicin Alternate drugs TMPSMZ, doxycycline, minocycline, a fluroquinolone, linezolid, quinpristindalfopristin

Correct Answer. a

Copyright 2014 Delhi Academy of Medical Sciences, All Rights Reserved.

29/71

(83).

Which of following is nicotinic antagonist?


a. Carbachol
b. Suxamethonium
c. epibatidine
d. Atracurium
Solution. Ans-83: (d) Atracurium
Ref:Read the text below
Sol:
Agonist Antagonist

Muscle-type:

Ganglion-type:

Neuromuscular
junction

autonomic ganglia

EPSP, mainly by
increased Na+ and
K+ permeability

acetylcholine[1]
carbachol
suxamethonium

-bungarotoxin[20]
-conotoxin
tubocurarine[1]
pancuronium
atracurium*

EPSP, mainly by
increased Na+ and
K+ permeability

acetylcholine[1]
carbachol
nicotine[1]
epibatidine
dimethylphenylpiperazinium
varenicline

mecamylamine[1][20]
trimetaphan
hexamethonium
bupropion
dextromethorphan
ibogaine
18-methoxycoronaridine

Correct Answer. d

(84).

Filgrastim is used in treatment of


a. Anemia
b. Neutropenia
c. Malaria
d. Filarial
Solution. Ans-84 : (b) Neutropenia
Ref:Read the text below
Sol:
Growth factors for treatment of cancer chemotherapy induced myelosuppression
1. G-CSF Filgrastim (Molgramostim)
2. GM-CSF Sargramostim
3. Erythropoietin Epoetin alfa and Darbopoietin alfa
4. For thrombocytopenia Oprelvekin (recombinant IL 11)
Uses of Filgrastim
1. It is used in treatment of severe neutropenia after autologous hematopoietic stem cell transplantation and highdose cancer chemotherapy
2. Neutropenia in patient with myelodysplasia
3. Neutropenia in AIDS patient receiving zidovudine

Correct Answer. b

Copyright 2014 Delhi Academy of Medical Sciences, All Rights Reserved.

30/71

(85).

Which is not the feature of digoxin toxicity?


a. Xanthopsia
b. Long PR interval
c. Long QT interval
d. Paroxysmal atrial tachycardia
Solution. Ans-85: (c) Long QT interval
Ref:Read the text below
Sol:
The pharmacological actions of digoxin usually results in electrocardiogram (ECG) changes, including ST depression or T wave inversion, which do not indicate toxicity.

PR interval prolongation, however, may be a sign of digoxin toxicity.


Additionally, increased intracellular Ca2+ may cause a type of arrhythmia called bigeminy (coupled beats), eventually ventricular tachycardia orfibrillation.
The combination of increased (atrial) arrhythmogenesis and inhibited atrio-ventricular conduction (for example paroxysmal atrial tachycardia with A-V block - so-called "PAT
with block") is said to be pathognomonic (i.e. diagnostic) of digoxin toxicity.
Vision changes are green/yellow halo around objects called Xanthopsia.
Correct Answer. c

(86).

True about oxytocin


a. Synthesised in posterior pituitary
b. Has ADH like effect
c. Is a ringed octapeptide
d. Bolus dose will cause increase in CVP
Solution. Ans-86: (b) Has ADH like effect
Ref:Read the text below
Sol:
Oxytocin is a nonapeptide.It is synthesised in hypothalamus but secreted by posterior pituitary.

Oxytocin has a slight antidiuretic action and causes water retention.


Correct Answer. b

(87).

With respect to sertonergic receptor action which of the following is true?


a. Metoclopromide is 5HT4 agonist
b. Ergotamine is 5HT1 antagonist
c. Ketanserin is 5HT2 agonist
d. Ondansetron is 5HT3 agonist
Solution. Ans-87: (a) Metoclopromide is 5HT4 agonist
Ref:Read the text below
Sol:
Ergotamine is 5HT1 agonist

Ketanserin is a 5HT2 antagonist


Ondansetron is 5HT3 antagonist
Correct Answer. a

Copyright 2014 Delhi Academy of Medical Sciences, All Rights Reserved.

31/71

(88).

Nesritide is
a. Brain natriuretic peptide analogue
b. Endothelin R antagonist
c. Gp IIb/IIIa antagonist
d. TNF alpha antagonist
Solution. Ans-88 : (a) Brain natriuretic peptide analogue
Ref:Read the text below
Sol:
Nesiritide
It is a recombinant human BNP analogue

Potent vasodilator, reduces ventricular filling pressure


Dose 2 g/kg IV
Adverse effect hypotension
Uses symptomatic heart failure even after treatment with diuretics and nitrates
Correct Answer. a

(89).

The leukotriene receptor antagonist used in bronchial asthma is


a. Zafirleukast
b. Zileuton
c. Ketotifen
d. Nedocromil
Solution. Ans-89 : (a) Zafirleukast
Ref:Read the text below
Sol:
Zafirleukast & Monteleukast - Leukotriene receptor antagonist

- Zileuton - Lipoxygenase inhibitor


- Nedocromil - Mast cell stabilizer
- Ketotifen - Mast cell stabilizer with anti histaminic (H1) property.
Correct Answer. a

(90).

Which of the following is a leukotriene antagonist?


a. Monteleukast
b. Zileuton
c. Sulotroban
d. Nedocromil
Solution. Ans-90: (a) Monteleukast
Ref:Read the text below
Sol:

- Montelukast and zafirlukast competitively antagonize cysLT1 receptor mediated bronchoconstriction, increased vascular permeability and recruitment of eosinophils.
- Parameters of lung function tests show variable but definite improvement.
- Montelukast and zafirlukast are both used for prophylactic therapy in asthma but they are not useful for terminating asthma episodes
- Both are very safe drugs. They produce few side effects like headache and rashes. Eosinophilia and neuropathy are infrequent. Few cases of Churg - Strauss syndrome have
been reported.
- They are well absorbed orally, highly plasma protein bound and metabolized by CYP29C (montelukast also by CYP3A4)

- The plasma t1/2 of montelukast is 3-6 hours, while that of zafirlukast is 8-12 hours.
Correct Answer. a

Copyright 2014 Delhi Academy of Medical Sciences, All Rights Reserved.

32/71

(91).

Not an anti epileptic


a. Phenytoin
b. Topiramate
c. Flunarazine
d. Carbamazepine
Solution. Ans-91: (c) Flunarazine
Ref:Read the text below
Sol:
Flunarazine is a weak calcium channel blocker claimed to be cerebroselective used in the prophylaxis of migraine.

Flunarizine is a relatively weak calcium channel blocker that also inhibits sodium channels
It is claimed to be as effective as propranolol, in prophylaxis of migraine but convincing proof is lacking
Side effects sedation, constipation, dry mouth, hypotension, flushing, weight gain and rarely extrapyramidal symptoms.
Correct Answer. c

(92).

Not true about tetracycline


a. Potentiates non-depolarising muscle relaxants
b. Inactivated by Ca+2 ion
c. Secreted in breast milk
d. Absorption is increased by magnesium trisilicate
Solution. Ans-92 : (d) Absorption is increased by magnesium trisilicate
Ref:Read the text below
Sol:
Side effects of tetracyclines
Tetracycline developing teeth (even when taken by the mother during pregnancy)

Can cause permanent teeth discoloration (yellow-gray-brown); infancy and childhood to eight (8) years old
Inactivated by Ca2+ ion, not to be taken with milk, yogurt, and other dairy products
Inactivated by aluminium, iron and zinc, not to be taken at the same time as indigestion remedies
Inactivated by common antacids and over-the-counter heartburn medicines.
Skin photosensitivity; exposure to the Sun or intense light is not recommended
Drug-induced lupus, and hepatitis
Can induce microvesicular fatty liver.
Tinnitus
May interfere with methotrexate by displacing it from the various protein binding sites
Can cause breathing complications as well as anaphylactic shock in some individuals
Should be avoided during pregnancy as it may affect bone growth of fetus.
Passes into breast milk and is harmful to breast-fed infants, and should therefore be avoided during breastfeeding if possible.

Correct Answer. d

Copyright 2014 Delhi Academy of Medical Sciences, All Rights Reserved.

33/71

(93).

Figure shows the quantal population doseresponse curves for the therapeutic and toxic effects of drugs X and Y. Both drugs are agonists
at the same receptor to produce the therapeutic response, and the maximum responses obtained with each agent are the same. The
toxicity curve in the figure shows the superimposed toxic response curves for drugs X and Y; they are identical in terms of the
concentration dependence. Which of the following statements is most correct?

a. At 1 X 105 M both drugs cause adverse effects in 90100% of patients.


b. Drug X has a larger therapeutic index than drug Y.
c. Drug X is more efficacious than drug Y.
d. Drug Y is more potent than drug X.
Solution. Ans-93: (b) Drug X has a larger therapeutic index than drug Y.
Ref: Read the text below
Sol :
Drug X has a larger therapeutic index (calculated as TD50/EC50) than drug Y. At a concentration of 1 105 M both drugs produce adverse effects in about 50%, not 90100%
of patients.
Drug X is not more efficacious than drug Y ; quantal doseresponse data provide no information about maximum efficacy and the question stem states that both drugs produce
the samemaximum responses.
Drug Yis not more potent than drug X ; it is less potent (it has a higher EC50).

Drug Y is not safer than drug X from the data presented; if anything, more adverse effects are produced by drug Y at its EC50 than by drug X at its EC50.
Correct Answer. b

(94).

Which of the following drugs inhibit osteoclastic activity more than osteoblastic activity and have been beneficial in osteoporosis,
hypercalcemia of malignancy, and Paget disease?
a. Alendronate
b. Calcitonin
c. Fluoride
d. Parathyroid hormone
Solution. Ans-94: (a) Alendronate
Ref: Read the text below
Sol :
Alendronate is a bisphosphonate, the class of drugs that has proved most effective in the treatment and prevention of osteoporosis.

This class of drug is also useful in the management of hypercalcemia and certain neoplasms.
Calcitonin has been shown to slow bone loss in postmenopausal women and to treat postoperative pancreatitis.
Fluoride is used primarily in the prevention of dental caries. Parathyroid hormone , marketed as Forteo, is currently used in the clinic to treat osteoporosis.
Correct Answer. a

Copyright 2014 Delhi Academy of Medical Sciences, All Rights Reserved.

34/71

(95).

The location of the lesion in Horner syndrome (either preganglionic or postganglionic) can be determined by the use of certain
sympathomimetics. Which of the following agents would distinguish between a preganglionic versus a postganglionic lesion?
a. Amphetamine
b. Atenolol
c. Epinephrine
d. Isoproterenol
Solution. Ans-95 : (a) Amphetamine
Ref: Read the text below
Sol :
Indirectly acting sympathomimetics (e.g., amphetamines, cocaine) are useful in this situation because their action requires the presence of intact postganglionic noradrenergic
neurons.
Thus administration of hydroxyamphetamine into the eye will cause mydriasis in a Horner patient if the lesion is preganglionic (postganglionic neuron intact), but not if the
lesion is postganglionic.
In contrast, directacting sympathomimetics will have the same effect regardless of the location of the lesion. Beta antagonists will not produce any effect on the pupil.

Correct Answer. a

(96).

A one-year-old male develops decreased breath sounds, and wheezing during a febrile episode, which is relieved by albuterol. What is the
mechanism of action of albuterol?
a. Inhibition of 5-lipoxygenase
b. Inhibition of mediator release
c. Inhibition of phosphodiesterase
d. Activation of -adrenergic receptors
Solution. Ans-96: (d) Activation of -adrenergic receptors
Ref: Read the text below
Sol :
Albuterol is a short-acting -adrenergic agonist. It is effective in obtaining immediate relief and is delivered by inhalation in acute episodes of bronchospasm.

Its action may last up to four hours.


Salmeterol is a long-acting -adrenergic agonist that can be taken orally and is useful for prophylaxis.
Although adrenergic agonists may be delivered orally, adverse effects are minimized when used by inhalation.
Correct Answer. d

(97).

A 61-year-old female has intermittent bouts of chest pain on exertion of two months duration, associated with numbness and tingling in
the fourth and fifth fingers of her left hand. An EKG is normal. She is placed on propranolol, which relieves her symptoms. What
cardiovascular effect did the drug have?
a. It decreased production of catecholamines
b. It dilated the coronary vasculature
c. It decreased the requirement for myocardial oxygen
d. It increased peripheral vascular resistance
Solution.
Ans-97: (c) It decreased the requirement for myocardial oxygen
Ref: Read the text below
Sol :
-adrenergic receptor blockers slow heart rate, lower blood pressure, and lessen cardiac contractility without reducing cardiac output; they also have a buffering action against
adrenergic stimulation of the cardiac autoregulatory mechanism.
These hemodynamic actions decrease the requirement of the heart for oxygen.

Correct Answer. c

Copyright 2014 Delhi Academy of Medical Sciences, All Rights Reserved.

35/71

(98).

The blood pressure of a 65-year-old male is well controlled by a Ca2+ channel blocker that is used to treat his essential hypertension.
When placed on cimetidine to control symptoms related to gastroesophageal reflux disease (GERD), he has episodes of dizziness. How
does cimetidines effect on Ca2+ channel blockers account for the dizziness?
a. It increases their rate of intestinal absorption
b. It decreases their plasma protein binding
c. It decreases their volume of distribution
d. It decreases their metabolism by cytochrome P450
Solution. Ans-98: (d) It decreases their metabolism by cytochrome P450
Ref: Read the text below
Sol :
Cimetidine slows the metabolism of Ca channel blockers, which are substrates for hepatic mixed-function oxidases.

Inhibition of cytochrome P450 activity is peculiar to cimetidine and is not a mechanism of action of other histamine 2 (H2) blockers.
Correct Answer. d

(99).

Which of the following is not associated with the ingestion of ethanol?


a. It is a hepatotoxic agent
b. It elevates body temperature
c. It suppresses the release of antidiuretic hormone
d. It can lead to gastritis and pancreatitis
Solution.
Ans-99 : (b) It elevates body temperature
Ref: Read the text below
Sol :
Ethanol is a CNS depressant. Among its many effects, it suppresses the release of antidiuretic hormone.

Ethanol also causes peripheral vasodilation, particularly of cutaneous blood vessels. Though this may give one a feeling of warmth, heat is being dissipated and body
temperature lowered.
Chronic use can lead to gastritis, pancreatitis, cirrhosis of the liver, and central effects such as Wernickes encephalopathy and Korsakoffs psychosis. Acute
overdose can lead to acidosis, hypoglycemia, and elevated intracranial pressure.

Correct Answer. b

(100).

A 25-year-old male with difficulty sleeping and poor appetite associated with weight loss is placed on amitriptyline. How is amitriptyline
classified?
a. As an MAOI
b. As a tricyclic nonselective amine reuptake inhibitor
c. As a heterocyclic nonselective amine reuptake inhibitor
d. As a selective serotonin reuptake inhibitor
Solution. Ans-100: (b) As a tricyclic nonselective amine reuptake inhibitor
Ref: Read the text below
Sol :
Amitriptyline is a tertiary amine tricyclic antidepressant. It functions as a norepinephrine reuptake inhibitor. Brain levels of amines are increased.

This results in increased vesicular stores of norepinephrine and serotonin.


Amitriptyline is a prototypical tricyclic antidepressant that has proved useful in patients with sleep and appetite disorders.
Correct Answer. b

Copyright 2014 Delhi Academy of Medical Sciences, All Rights Reserved.

36/71

(101).

A patient with intractable itching would best respond to which of the following?
a. Chlorpromazine
b. Pimozide
c. Haloperidol
d. Risperidone
Solution. Ans-101: (a) Chlorpromazine
Ref: Read the text below
Sol :
Agents with H1 receptor blocking actions are effective in reducing itching. H1 receptor blockade is typical of phenothiazines with short side chains.

Correct Answer. a

(102).

A 26-year-old female with reactive depression complains of missing her period and having milk discharge from her breasts. She has no
signs of pregnancy, including a negative pregnancy test. Which of the following might have caused these findings?
a. Clomipramine
b. Amoxapine
c. Fluoxetine
d. Mirtazapine
Solution. Ans-102: (b) Amoxapine
Ref: Read the text below
Sol :
Amoxapine is a heterocyclic antidepressant that has effects on norepinephrine and serotonin uptake. It is useful in psychotic patients who are depressed.

The dopaminergic antagonism caused by amoxapine may lead to the amenorrheagalactorrhea syndrome.
Correct Answer. b

(103).

All of the following drugs are used topically in the treatment of chronic wide-angle glaucoma. Which of these agents reduces intraocular
pressure by decreasing the formation of the aqueous humor?
a. Timolol
b. Echothiophate
c. Pilocarpine
d. Isofluorphate
Solution. Ans-103: (a) Timolol
Ref: Read the text below
Sol :
When applied topically to the eye, both the direct-acting cholinomimetic agents (e.g., pilo- carpine) and those cholinomimetic drugs that act by inhibition of AChE (e.g.,
echothiophate, isofluorphate, and physostigmine) cause miosis by contracting the sphincter muscle of the iris and reducing ocular pressure by contracting the ciliary muscle.
In patients with glaucoma, this latter effect permits greater drainage of the aqueous humor through the trabecular meshwork in the canal of Schlemm and a reduction in
resistance to outflow of the aqueous humor. Certain -adrenergic blocking agents (e.g., timolol and levobunolol) applied to the eye are also very useful in treating chronic wideangle glaucoma.
These drugs appear to act by decreasing the secretion (or formation) of the aqueous humor by antagonizing the effect of circulating catecholamines on -adrenergic receptors
in the ciliary epithelium.

Correct Answer. a

Copyright 2014 Delhi Academy of Medical Sciences, All Rights Reserved.

37/71

(104).

Neostigmine will effectively antagonize skeletal muscle relaxation produced by


a. Pancuronium
b. Succinylcholine
c. Diazepam
d. Baclofen
Solution. Ans-104: (a) Pancuronium
Ref: Read the text below
Sol :
Anticholinesterase agents, such as neostigmine, will delay the catabolism of ACh that is released from parasympathetic autonomic and somatic nerve terminals.

At the neuromuscular junction, this results in increased competition for the NM receptors by ACh (the agonist) and the curariform drugs (the antagonists) such as tubocurarine
and pancuronium. In addition, neostigmine has a direct stimulating action on the skeletal muscle junction, which enhances its ability to antagonize the competitive neuromuscular
blockers.
The activity of succinylcholine at the neuromuscular junction will be exacerbated by neostigmine, because succinylcholine is inactivated by AChE. The skeletal muscle
relaxation that may result from toxic doses of nicotine-blocking NM receptors will be unaffected by neostigmine.
Diazepam and baclofen are centrally acting skeletal muscle relaxants whose effects are not altered by the peripheral actions of neostigmine.

Correct Answer. a

(105).

Epinephrine may be mixed with certain anesthetics, such as procaine, in order to


a. Stimulate local wound repair
b. Promote hemostasis
c. Retard their systemic absorption
d. Facilitate their distribution along nerves and fascial planes
Solution.
Ans-105: (c) Retard their systemic absorption
Ref: Read the text below
Sol :
The addition of a vasoconstrictor, such as epinephrine or phenylephrine, to certain short-acting, local anesthetics is a common practice in order to prevent the rapid systemic
absorption of the local anesthetics, to prolong the local action, and to decrease the potential systemic reactions.
Some local anesthetics cause vasodilation, which allows more compound to escape the tissue and enter the blood.

Procaine is an ester-type local anesthetic with a short duration of action due to rather rapid biotransformation in the plasma by cholinesterases.
The duration of action of the drug during infiltration anesthesia is greatly increased by the addition of epinephrine, which reduces the vasodilation caused by procaine.
Correct Answer. c

(106).

Both phentolamine and prazosin


a. Are competitive antagonists at 1-adrenergic receptors
b. Have potent direct vasodilator actions on vascular smooth muscle
c. Enhance gastric acid secretion through a histamine-like effect
d. Cause hypotension and bradycardia
Solution. Ans-106: (a) Are competitive antagonists at 1-adrenergic receptors
Ref: Read the text below
Sol :
Phentolamine is a nonselective -adrenergic receptor blocker (i.e., it has affinity for both 1- and 2-adrenergic receptor sites). It also has a prominent direct relaxant
(musculotropic spasmolytic) effect on arterioles, which results in vasodilation and reflex tachycardia.
In addition, phentolamine can block the effects of serotonin and will increase hydrochloric acid and pepsin secretion from the stomach. Phentolamine is used for the short-term
control of hypertension in patients with pheochromocytoma (i.e., a type of secondary hypertension); because of the high incidence of tachycardia associated with the compound, it
is not used chronically for the treatment of essential hypertension.
Prazosin is a selective 1-adrenergic receptor antagonist that, at therapeutic doses, has little activity at 2-adrenergic receptors and clinically insignificant direct vasodilating
activity. The drug does not cause the other effects attributed to phentolamine. Most important, it produces less tachycardia than does phentolamine and, therefore, is useful in the
treatment of essential hypertension.

Correct Answer. a

Copyright 2014 Delhi Academy of Medical Sciences, All Rights Reserved.

38/71

(107).

A 10-year-old male displays hyperactivity and is unable to focus on his schoolwork because of an inability to focus on the activity. Which
of the following might prove effective in this patient?
a. Methylphenidate
b. Terbutaline
c. Dobutamine
d. Pancuronium
Solution. Ans-107: (a) Methylphenidate
Ref: Read the text below
Sol :
Methylphenidate is similar to amphetamine and acts as a CNS stimulant, with more pronounced effects on mental than on motor activities.

It is effective in the treatment of narcolepsy and attention-deficit hyperactivity disorders.


Correct Answer. a

(108).

Nicotine in low doses may cause


a. Decreased tone and motor activity of the small intestine
b. Stimulation of the respiratory rate and depth
c. Miosis
d. Bradycardia
Solution. Ans-108: (b) Stimulation of the respiratory rate and depth
Ref: Read the text below
Sol :
Nicotine is a depolarizing ganglionic blocking agent; that is, it stimulates nicotinic receptors in low doses and predominantly blocks at high-dose levels.

The effect of nicotine on a particular tissue or organ depends on the relative contribution to the function made by each division of the ANS.
The effects on the cardiovascular system are complex. Stimulation of the cardiac vagal ganglia causes bradycardia. This is countered by sympathetic stimulation to the heart
(tachycardia), blood vessels (vasoconstriction), and adrenal medulla (catecholamine release: tachycardia and vasoconstriction). Thus, the net effectof nicotine on the heart is
tachycardia, not bradycardia.
Low doses of nicotineaugment respiration by excitation of the chemoreceptors of the carotid body and aortic arch. Higher doses also stimulate the medullary respiratory center
and increase respiration through CNS activity.
Large amounts of nicotine cause respiratory failure from medullary paralysis and blockade of the skeletal muscles of respiration.

Correct Answer. b

(109).

A naturally occurring substance that is useful in treating Pagets disease of the bone is
a. Etidronate
b. Cortisol
c. Calcitonin
d. Parathyroid hormone (PTH)
Solution. Ans-109 : (c) Calcitonin
Ref: Read the text below
Sol :
Calcitonin is useful in the therapy of Pagets disease of bone (osteitis deformans). Calcitonin therapy reduces urinary hydroxyproline excretion and serum alkaline phosphatase
activity and provides some symptomatic relief.
Presumably, these effects result from the ability of calcitonin to inhibit bone resorption. Side effects of long-term therapy with this hormone can include nausea, edema of the
hands, and urticaria. The appearance of neutralizing antibodies may explain the development of resistance to treatment.
Etidronate is a synthetic drug that is useful in Pagets disease. The compound is orally effective and lacks the antigenicity associated with calcitonin.

Correct Answer. c

Copyright 2014 Delhi Academy of Medical Sciences, All Rights Reserved.

39/71

(110).

A patient becomes markedly tetanic following a recent thyroidectomy. This symptom can be rapidly reversed by the administration of
a. Vitamin D
b. Calcitonin
c. Plicamycin (mithramycin)
d. Calcium gluconate (CaG)
Solution. Ans-110: (d) Calcium gluconate (CaG)
Ref: Read the text below
Sol :
Administration of intravenous CaG would immediately correct the tetany that might occur in a patient in whom a thyroidectomy was recently performed.

Parathyroid hormone would act more slowly but could be given for its future stabilizing effect.
Long-term control of a patient after a thyroidectomy can be obtained with vitamin D and dietary therapy.
Calcitonin is a hypocalcemic antagonist of parathyroid hormone. Plicamycin (mithramycin) is used to treat Pagets disease and hypercalcemia. The dose employed is about onetenth the amount used for plicamycins cytotoxic action.

Correct Answer. d

(111).

A 55-year-old postmenopausal female develops weakness, polyuria, and polydipsia. Nephrocalcinosis is detected by a computed
tomography (CT) scan. Her serum creatinine is elevated. Which of the following agents may have caused these adverse effects?
a. Estrogens
b. Prednisone
c. Etidronate
d. Vitamin D
Solution. Ans-111: (d) Vitamin D
Ref: Read the text below
Sol :
Enthusiastic overmedication with vitamin D may lead to a toxic syndrome called hypervitaminosis D.

The initial symptoms can include weakness, nausea, weight loss, anemia, and mild acidosis.
As the excessive doses are continued, signs of nephrotoxicity are manifested, such as polyuria, polydipsia, azotemia, and eventually nephrocalcinosis. In adults, osteoporosis
can occur. Also, there is CNS impairment, which can result in mental retardation and convulsions.

Correct Answer. d

(112).

Of the following, which drug does not increase the need for insulin?
a. Epinephrine
b. Hydrocortisone
c. Dexamethasone
d. Ethanol (acute ingestion)
Solution. Ans-112: (d) Ethanol (acute ingestion)
Ref: Read the text below
Sol :
The regulation of levels of blood glucose by insulin and the general effectiveness of insulin are altered with the coadministration of other drugs.

Epinephrine enhances glycogenolysis and thereby elevates glucose in the plasma. Glucocorticoids (e.g., hydrocortisone and dexamethasone) stimulate gluconeogenesis, reduce
the peripheral utilization of glucose, and decrease the sensitivity of tissues to insulin.
Chlorthalidone, a thiazide-related diuretic, may induce hyperglycemia by inhibition of the release of insulin and decreased use of glucose by peripheral tissues. In the presence
of ethanol, the effect of insulin is enhanced. When ethanol is acutely ingested in sufficient quantities, the drug causes an alteration in carbohydrate metabolism that results in
hypoglycemia.
The exact mechanism of the hypoglycemic effect of ethanol is not known.

Correct Answer. d

Copyright 2014 Delhi Academy of Medical Sciences, All Rights Reserved.

40/71

(113).

A 60-year-old male alcoholic treated for type II diabetes mellitus develops lactic acidosis. Which of the following oral hypoglycemic
agents might cause this adverse effect?
a. Glyburide
b. Metformin
c. Acarbose
d. Rosiglitazone
Solution. Ans-113: (b) Metformin
Ref: Read the text below
Sol :
Metformin is contraindicated in patients with type II diabetes in a number of instances, including renal disease, liver disease, chronic cardiopulmonary dysfunction leading to
hypoxia, and alcoholism.

Correct Answer. b

(114).

Which of the following is true regarding Clopidogrel


a. Is an isomer of ticlopidine
b. Is a thienopyridine
c. Causes neutrophilia
d. Has reversible inhibhition of ADP receptor.
Solution. Ans-114: (b) Is a thienopyridine
Ref:Read the text below
Sol:
Clopidogrel is a oral theinopyridine(ie. ADP receptor/P2y12 inhibhitor) antiplatelet agent.It blocks activation of glycoprotein IIb/IIIa pathway.it irreversibly inhibhits P2Y12
subtype ADP receptor.
It is used to prevent vascular ischemic events in patients with symptomatic atherosclerosis, prevention of acute coronary syndrome without ST-segment elevation(NSTE-MI)
and St elevation MI.
It causes severe neutropenia, not neutrophilia.

Correct Answer. b

(115).

Not true for tamoxifen


a. Competitive partial agonist-inhibitor of estrogen
b. Suppresses serum levels of insulin-like growth factor-1
c. Downregulates local TGF-beta production
d. More effective in oopherectomised women.
Solution. Ans-115: (c) Downregulates local TGF-beta production
Ref:Read the text below
Sol:
Tamoxifen is a SERM used as a anticancer agent.
Mechanism of Action:
Competitive partial agonist-inhibitor of estrogen

Binds to estrogen-sensitive tissues (receptors present)


Best antiestrogen effect requires minimal endogenous estrogen presence {estradiol has a much higher affinity for the estrogen receptor than tamoxifen's affinity for the
estrogen receptor}
Suppresses serum levels of insulin-like growth factor-1; and up-regulates local TGF-beta production. These properties may explain tamoxifen antitumor activity in melanoma
and ovarian cancer.

Correct Answer. c

Copyright 2014 Delhi Academy of Medical Sciences, All Rights Reserved.

41/71

(116).

One of the following is a prodrug,


a. Enalapril
b. Neostigmine
c. Esmolol
d. Captopril
Solution. Ans-116: (a) Enalapril
Ref:Read the text below
Sol:

Correct Answer. a

(117).

DENOSUMAB a monoclonal antibody against RANKL receptor is used in treatment of,


a. Rheumatoid arthritis
b. Osteoporosis
c. Osteoarthritis
d. SLE
Solution. Ans-117: (b) Osteoporosis
Ref:Read the text below
Sol:
DENOSUMAB is designed to target RANKL ligand, a protein that acts as the primary signal to promote bone removal. In many bone loss conditions, RANK ligand overwhelms
the bodys natural defence mechanism against bone destruction.
Pre clinical models have demonstrated that inhibiting RANK ligand, leads to significant improvements in cortical cular bone density, volume and strength.

Denosumab is used to treat primary osteoporosis in post menopausal women.


It is also tried in treatment of drug induced bone loss, bone metastases, multiple myeloma and rheumatoid arthritis.
Correct Answer. b

(118).

Bevirimat is a new antiretroviral drug.It is a


a. Protease inhibhitor
b. NRTI
c. NNRTI
d. Virocidal
Solution. Ans-118: (a) Protease inhibhitor
Ref:Read the text below
Sol:
Like protease inhibitors, bevirimat and other maturation inhibitors interfere with protease processing of newly translated HIV polyprotein precursor, calledgag.

This molecule contains a number of HIV proteins in a single polypeptide which is then cleaved by the enzyme protease to produce functional structural proteins.
However, unlike the protease inhibitors, bevirimat binds the gag protein, not protease.Of the given options this is the best answer.
Correct Answer. a

Copyright 2014 Delhi Academy of Medical Sciences, All Rights Reserved.

42/71

(119).

Which of the following is not a antiprogestin?


a. Mifepristone
b. Mepitiostane
c. Asoprisnil
d. Ulipristal acetate
Solution. Ans-119: (b) Mepitiostane
Ref:Read the text below
Sol:
Mepitiostane is a anti-estrogen drug,Rest all are antiprogesterones.

Correct Answer. b

(120).

GABA receptor antagonist is


a. Benzodiazepines
b. Zolpidem
c. Bicuculline
d. Muscimol
Solution. Ans-120: (c) Bicuculline
Ref:Read the text below
Sol:
Bicuculine
Bicuculline is a light-sensitive competitive antagonist of GABAA receptors.

The action of bicuculline is primarily on the ionotropic GABAA receptors, which are ligand-gated ion channels concerned chiefly with the passing of chloride ions across
the cell membrane, thus promoting an inhibitory influence on the target neuron.
These receptors are the major targets for benzodiazepines and related anxiolytic drugs.
Correct Answer. c

(121).

The following drugs are used in obesity except,


a. Orlistat
b. Sibutaramine
c. Olestra
d. Neuropeptide Y analogues
Solution. Ans-121: (d) Neuropeptide Y analogues
Ref:Read the text below
Sol:
Neuropeptide Y antagonists and not analogues are used for obesity

Correct Answer. d

Copyright 2014 Delhi Academy of Medical Sciences, All Rights Reserved.

43/71

(122).

A 44 year old male with Child's grade C cirrhosis presented with haematemesis. Which one of the following drugs, administered
intravenously, would be the most appropriate, immediate, treatment?
a. Isosorbide dinitrate
b. Omeprazole.
c. Propranolol
d. Somatostatin
Solution. Ans 122: (d) Somatostatin
Reference Read the text below
Sol:
The suggestion is that this patient is at particularly high risk of oesophageal varices. Childs classification of cirrhosis is a points scale based upon ascites/bilirubin etc reflecting
prognosis.
Graded depending upon the points scored from A-C with C reflecting greatest risk.

Somatostatin acts to reduce portal pressures and has been demonstrated to be as effective as endoscopy at controlling variceal bleeding in the acute setting. Beta-blockers can
be used as oral prophylaxis for oesophageal varices.
IV Omeprazole has also been shown to be effective in reducing mortality in GI haemorrhage of any cause but somatostatin may be expected to be superior for the above patient.

Correct Answer. d

(123).

An 18 year-old woman presents with an acute pulmonary embolism in the ninth week of pregnancy.What is the most appropriate
treatment for this patient throughout her pregnancy?
a. Aspirin
b. Intravenous unfractionated heparin
c. Subcutaneous low molecular weight heparin
d. Subcutaneous unfractionated heparin
Solution. Ans 123: (c) Subcutaneous low molecular weight heparin
Reference Read the text below
Sol:
Anticoagulation with subcutaneous heparin is recommended in most guidelines with LMWH being a suitable alternative.

The latter has the advantage of requiring no monitoring but is a less well established therapy in pregnancy.
She cannot be treated with IV heparin throughout her pregnancy and Warfarin is also contra-indicated due to teratogenicity.
Aspirin provides no demonstrable prophylactic value for venous thromboembolism.
Correct Answer. c

(124).

The difference between hyoscine and atropine is that hyoscine.


a. Exerts depressant effects on the CNS at relatively low doses
b. Exerts more potent effects on the heart than on the eye
c. Is longer acting
d. Has weaker animation sickness activity.
Solution. Ans 124: (a) Exerts depressant effects on the CNS at relatively low doses
Reference Read the text below
Sol:
One peculiar response to atropine is the bradycardia reportedly seen with small doses (eg. 0.4 mg in an adult) - the mechanism of this paradoxical effect is still not clear

Hyoscine exerts depressant effects on the CNS at relatively low doses


Scopolamine has pronounced CNS sedative effects, and may also be useful in preventing nausea and vomiting, especially due to motion sickness
Correct Answer. a

Copyright 2014 Delhi Academy of Medical Sciences, All Rights Reserved.

44/71

(125).

Intravenous furosemide is used for quick relief of dyspnoea in acute left ventricular failure by
a. Producing bronchodilation
b. Causing rapid diuresis and reducing circulating blood volume
c. Causing venodilation
d. Stimulating left ventricular contractility
Solution. Ans 125: (c) Causing venodilation
Reference Read the text below
Sol:
Intravenous furosemide is used for rapid control of symptoms in acute left ventricular failure.

It provides quick relief of dyspnoea by exerting venodilation.


Correct Answer. c

(126).

Concomitant use of ketorolac and aspirinis contraindicated because of increased risk of:
a. Vomiting.
b. Myelopathy.
c. GI bleed.
d. Cardiomyopathy.
Solution. Ans 126: (c) GI bleed
Reference Read the text below
Sol:
Concomitant use of ketorolac and aspirinis contraindicated because of increased risk of GI bleeding and platelet aggregation inhibition.

Children who have received live varicella virus vaccine should avoid aspirin for at least 6 weeks after vaccination to prevent Reye's syndrome.
Correct Answer. c

(127).

M1 receptors are mainly found in the:


a. GIT.
b. Liver.
c. Brain.
d. kidney
Solution. Ans 127: (c) Brain.
Reference Read the text below
Sol:
The five muscarinic receptors
Although five muscarinic receptors have been identified, helpfully labelled M1 to M5, only three are well-characterised. (M4 receptors
are confined to the CNS, and M5 are not well understood). Here are the three important ones:
+
1. M1 receptors are mainly found in the nervous system - they mediate excitatory effects, lowering transmembrane potential by a decrease in K ion conductance; as an added
wrinkle, they mediate increased gastric acid secretion seen with vagal stimulation. M1 receptors work via phospholipase C, increasing IP3 and DAG levels;
2. M2 mediate the cardiac effects of (for example) vagal stimulation - they are inhibitory (hyperpolarizing membranes by increasing potassium conductance). M2 receptors are
also found presynaptically in a variety of situations - this fits in with their inhibitory nature, here exploited for negative feedback. M2 receptors lower intracellular cAMP levels
(via Gk );
3. M3 receptors are responsible for all the other effects of parasympathetic stimulation, as they are the cholinergic excitatory receptors found on glands and smooth muscle. In
our first tutorial, we detailed all the various effects of such receptor stimulation. M3 receptors are similar to M1 in their use of phospholipase C. Physiology is however never
simple - vascular smooth muscle relaxes in some situations due to M3 receptor stimulation. This relaxation is mediated by endothelial release of nitric oxide (NO), and occurs in
some vascular beds that appear devoid of parasympathetic innervation.
4. M5 receptors seem similar to M1 and M3 in their effects; M4 are similar to M2.

Correct Answer. c

Copyright 2014 Delhi Academy of Medical Sciences, All Rights Reserved.

45/71

(128).

The drug effective for treatment as well as prophylaxis of angina pectoris is


a. Isosorbide dinitrate
b. Pentaerythritol tetranitrate
c. Diltiazem
d. Dipyridamole
Solution.
Ans 128: (a) Isosorbide dinitrate
Reference Read the text below
Sol:
Isosorbide dinitrate is a nitrate used pharmacologically as a vasodilator, eg. in angina pectoris but also for anal fissure, a condition which is known to involve decreased blood
supply leading to poor healing.
An antianginal agent. Its prototype is nitrogycerin.
Indications It is prescribed as a coronary vasodilator in the treatment of angina pectoris and congestive heart failure and esophageal spasm caused by GI reflux.
Contraindications Closed-angle glaucoma, known hypersensitivity to this drug, concurrent use of drugs for erectile dysfunction, narrow-angle glaucoma, head trauma, or severe
anemia prohibits its use.
Adverse effects The most serious effect is occasional marked hypotension. Flushing, headache, lightheadedness, and dizziness may also occur.

Correct Answer. a

(129).

The drug of choice for obsessive compulsion is


a. Amphetamine
b. Diazepam
c. Clomipramine
d. Chlorpromazine
Solution. Ans 129: (c) Clomipramine
Reference Read the text below
Sol:
DRUG TREATMENT FOR OBSESSIVE COMPULSION
Some patients may improve dramatically with specific serotonin reuptake inhibitors (SSRIs) Clomipramine (75-300 mg/day), a non specific serotonin reuptake inhibitory (SRI),
is the first drug used effectively.
Antidepressant with specific serotonin reuptake inhibitors (SSRIs) like clomipramine (75-300 mg/day) is effective in treatment of OCD. It can be given orally or parenterally.

Fluoxetine (20-80 mg/day) is the second drug of choice while paroxetine, fluvoxamine and sertraline are also effective in some patients.
Correct Answer. c

(130).

A 43 year old woman with atopic dermatitis (atopic eczema) presented with an acute generalized exacerbation of her disease. She was
admitted to hospital but failed to improve with emollients, topical betamethasone-17-valerate and oral antihistamine. Which one of the
following drugs is the most appropriate treatment?
a. Acitretin
b. Amoxycillin
c. Ciclosporin
d. Colchicine
Solution. Ans 130: (c) Ciclosporin
Reference Read the text below
Sol:
Cycloporin is a well used drug in the treatment of atopic dermatitis. It is usually at doses of 2-5 mg/kg.

The pathophysiology of AD is complex but the T lymphocytes are involved and it is known that there is an increased production of cytokines particularly IL-4.
Ciclosporin is a suppressor of T cells and in that respect works very well in atopicdermatitis and psoriasis.
The side effects of hypertension and renal toxicity limit its use. These patients are seen monthly to have their BP and U+Es checked.
Correct Answer. c

Copyright 2014 Delhi Academy of Medical Sciences, All Rights Reserved.

46/71

(131).

The action of noradrenaline released at sympathetic nerve endings is terminated by


a. Enzymatic decarboxylation
b. Enzymatic inactivation by catechol-O-methyl transferase
c. Re-uptake of noradrenaline by the axonal terminals
d. Oxidative deamination by monoamine oxidase
Solution. Ans 131: (c) Re-uptake of noradrenaline by the axonal terminals
Reference Read the text below
Sol:
The effects of neurotransmitter release are principally terminated by neuronal uptake.

Intraneuronal NA is usually taken back up into the neurosecretory granules and a small amount is metabolised by MAO.
Even smaller quantities that escape into the circulation are metabolised by COMT.
Correct Answer. c

(132).

Figure illustrates a current concept of the control of gastric acid secretion. Which of the following drugs acts at the site labeled
ATPase?

a. Aluminum hydroxide
b. Misoprostol
c. Omeprazole
d. Ranitidine
Solution. Ans-132: (c) Omeprazole
Ref: Read the text below
Sol :
Omeprazole is a proton pump inhibitor (like lansoprazole, esomeprazole, pantoprazole,
and rabeprazole) and irreversibly inhibits the H+/K+-ATPase proton pump. Proton pump inhibitors are the most efficacious agents available for the treatment of acid-peptic
disease.
Aluminum hydroxide is an antacid that may reduce symptoms of peptic disease, but is inconvenient to use and constipating. Misoprostol is an orally active PGE1 analog that is
used in the treatment or prevention of NSAID-induced ulcers. Ranitidine is an H2-receptor antagonist.
Gastric acid secretion involves activation of H2-receptors, and peptic disorders such as duodenal ulcer often respond to treatment with H2-receptor antagonists. Sucralfate ) is a
sucrosealuminum- sulfate compound that polymerizes in an acid environment and is able to form a protective coating over an ulcer bed.
It must be taken four times a day, so it is less convenient than H2-blockers or proton pump inhibitors.

Correct Answer. c

Copyright 2014 Delhi Academy of Medical Sciences, All Rights Reserved.

47/71

(133).

A 35 year old woman with alcoholic cirrhosis is admitted with deteriorating encephalopathy and abdominal discomfort. An ascitic tap
revealed a polymorphonuclear cell count of 350 cells per mm3.Which of the following is the most appropriate therapy?
a. Intravenous amoxicillin
b. Intravenous cefotaxime
c. Intravenous metronidazole
d. Oral neomycin
Solution. Ans 133: (b) Intravenous cefotaxime
Reference Read the text below
Sol:
This lady has Spontaneous Bacterial Peritonitis as suggested by the typical history, ascites and raised polymorphonuclear count within the ascitic tap.

It is most commonly seen in alcoholic cirrhosis and the causative organism is usually E. Coli, Klebsiella, S Pneumoniae or Enterococci. (Compare this with the mixed growth
seen in other forms of peritonitis).
Sending some ascitic fluid in blood culture bottles increases the yield. Initial treatment is with broad spectrum antibiotics such as cefotaxime.

Norfloxacin is recommended for short term prophylaxis.


Correct Answer. b

(134).

With which of the following is hyperprolactinaemia associated?


a. Cabergoline therapy
b. Depression
c. Fluoxetine therapy
d. Hyperthyroidism
Solution. Ans 134: (c) Fluoxetine therapy
Reference Read the text below
Sol:
Hyperprolactinaemia may be manifest by a milky discharge from the breasts.

Causes include, prolactinoma, hypothyroidism (far increased TRH), Non-functional tumour with stalk compression and drugs in particular dopamine antagonists such as
chlorpromazine, haloperidol and domperidone.
Pregnancy is a particularly common cause of hyperprolactinaemia.

Other drugs that are occasionally reported include SSRIs. PCOs is often associated with idiopathic hyperprolactinaemia.
Correct Answer. c

(135).

A 60 year old man has Parkinson's disease. He is started on treatment with L-dopa and dopa decarboxylase inhibitor therapy. However
he continues to have troublesome tremor. Which of the following drugs would be most likely to help?
a. Amantadine.
b. Benzexol.
c. Propranolol
d. Ropinirole
Solution.
Ans 135: (b) Benzexol.
Reference Read the text below
Sol:
Anticholinergic drugs such as Benzexol remains the treatment of choice in Parkinsonian tremor. L-dopa, selegiline and dopamine agonists are less effective in tremor.

Propranolol is the treatment of choice in essential tremor.


Correct Answer. b

Copyright 2014 Delhi Academy of Medical Sciences, All Rights Reserved.

48/71

(136).

Which one of the following enzymes does Selegiline act on to cause this adjuvant action?
a. Catechol-0-methyltransferase
b. Dopa decarboxylase
c. Dopamine hydroxylase
d. Monoamine oxidase
Solution. Ans 136: (d) Monoamine oxidase
Reference Read the text below
Sol:
Selegiline is a MAO-B inhibitor.

Correct Answer. d

(137).

A 30 year old male presented with a paranoid psychosis accompanied by visual hallucinations which resolved over the next three days.
Which one of the following is the most likely diagnosis?
a. Alcohol withdrawal
b. Diazepam dependence.
c. Fluoxetine overdose.
d. Heroin withdrawal
Solution. Ans 137: (a) Alcohol withdrawal
Reference Read the text below
Sol:
The paranoid psychosis with visual hallucinations is highly suggestive of delirium tremens alcohol withdrawal.

Correct Answer. a

(138).

Which of the following concerning diamorphine elixir for the relief of pain in terminal patients is correct?
a. Analgesia is enhanced if cocaine is added
b. Constipation is a characteristic sequel to treatment
c. Dependence occurs rapidly
d. Initial sedation typically continues whilst the drug is administered
Solution. Ans 138: (b) Constipation is a characteristic sequel to treatment
Reference Read the text below
Sol:
Sedation occurring in the first few days typically wears off, leaving the patient alert.

Hallucinations also tend to occur.


An aperient should always be added to the treatment regime.
An intramuscual injection is three times more effective than the same oral dose (Cornwall Trainers)
Correct Answer. b

Copyright 2014 Delhi Academy of Medical Sciences, All Rights Reserved.

49/71

(139).

Which of the following antiarrhythmic drugs may be used in the treatment of long QT syndrome?
a. Amiodarone
b. Atenolol
c. Flecainide
d. Propofanone
Solution. Ans 139: (b) Atenolol
Reference Read the text below
Sol:
Betablockers are the mainstay of treatment in long QT syndrome.

The most commonly used drugs are propranolol and nadolol but metoprolol and atenolol are also used.
Implantable Cardioverter-Defibrillators are the most effective treatment in high risk cases.
The others drugs may produce a prolongation of the QT interval exacerbating risk of polymorphic VT and Torsades de pointes.
Correct Answer. b

(140).

A 45 year old male type 1 diabetic with a number of complex diabetic gastrointestinal complications is noted to have a PR interval of
0.18s, a QRS duration of 0.1s and a QT interval of 0.48s on routine ECG. Which of the following drugs may be responsible?
a. Cisapride
b. Octreotide
c. Co-trimoxazole
d. Domperidone
Solution. Ans 140: (a) Cisapride
Reference Read the text below
Sol:
Cisapride has been withdrawn due to the problem of prolonged QT interval and torsades de pointe.

Prolonged QT is defined as greater than 0.45s.


Other agents include amitriptyline and pheonthiazines yet metoclopramide and dompaeridone are not associated.
Correct Answer. a

(141).

A 64-year-old man presents with difficulty urinating and dribbling at the end of urination.Which of the following drugs would be most
appropriate for treating this mans conditions?
a. Finasteride
b. Leuprolide
c. Mifepristone
d. Pergolide
Solution. Ans-141 : (a) Finasteride
Ref:Read the text below
Sol:
Finasteride, an inhibitor of 5-alpha-reductase, prevents the conversion of testosterone to dihydrotestosterone (DHT).

Because dihydrotestosterone is essential for the normal growth and development of the prostate gland, finasteride is an effective treatment for benign prostatic hyperplasia,
which is a DHT-dependent process.

Correct Answer. a

Copyright 2014 Delhi Academy of Medical Sciences, All Rights Reserved.

50/71

(142).

A 33-year-old man receiving chemotherapy for testicular carcinoma develops signs of renal tubular damage. Which of the following drugs
is most likely responsible for this nephrotoxicity?
a. Bleomycin
b. Cisplatin
c. Cyclophosphamide
d. Vinblastine
Solution. Ans-142: (b) Cisplatin
Ref:Read the text below
Sol:
Cisplatin is an antineoplastic drug used in the treatment of carcinoma of the testes (along with bleomycin and vinblastine) ovaries, bladder, and lung (especially small cell).

Along with the typical side effects of nausea, vomiting, and bone marrow suppression, cisplatin is notable for its dose-limiting nephrotoxicity and ototoxicity.
Correct Answer. b

(143).

All of the following are inhibitors of glucagon secretion except


a. Insulin
b. Hyperglycemia
c. Somatostatin
d. Amino acids.
Solution. Ans-143: (d) Amino acids.
Ref:Read the text below
Sol:
Amino acids stimulate glucagon secretion. The effects of glucose on glucagon secretion are reciprocal to those of insulin secretion in that hyperglycemia suppresses the
secretion of glucagon.
Insulin is a potent inhibitor of glucagon secretion and acts within the pancreatic islet.

Catecholamines and glucocorticoids, such as cortisol, inhibit glucagon secretion.


Correct Answer. d

(144).

A continuous IV infusion of lidocaine is given to a 70-kg patient with cardiac arrhythmias. The pharmacokinetic parameters for lidocaine
are as follows clearance (CL) = 9 mL/min/Kg, volume of distribution (Vd) = 70 L, half-life = 2 hours. How long will it take for drug levels
to reach 87.5% of steady state?
a. 1.75hours
b. 3.5hours
c. 5.5hours
d. 6.0hours
Solution. Ans-144: (d) 6.0hours
Ref:Read the text below
Sol:
It takes one half-life to reach 50% of steady state, two half-lives to reach 75% of steady state, three half-lives to reach 87.5% of steady state, and four half-lives to reach 93.75%
of steady state.
Each successive half-life brings the level of the drug closer to 100% but by a smaller amount (half of the previous increase) in each case. In this case, it should take three halflives to reach 87.5% of steady state, or 2x3=6 hours.

Correct Answer. d

Copyright 2014 Delhi Academy of Medical Sciences, All Rights Reserved.

51/71

(145).

A 5-year-old boy accidentally ingested a large dose of rat poison. He is conscious but appears quite agitated. On physical examination, his
blood pressure is 110/70 mm Hg and pulse is 90/min. Laboratory results are signification for an elevated prothrombin time (PT) but a
normal partial prothrombin time (PTT). Which of the following is the most appropriate pharmacotherapy?
a. Atropine
b. Flumazenil
c. N-acetlcysteine
d. Vitamin K
Solution. Ans-145: (d) Vitamin K
Ref:Read the text below
Sol:
As you might have guessed from his elevated PT, the active ingredient in rat poison is warfarin.

It acts as an anticoagulant by interfering with the normal hepatic synthesis of the vitamin K-dependent clotting factors II, VII, IX, and X.
The most important adverse effect of warfarin is bleeding. The action of warfarin can be reversed with vitamin K.
Correct Answer. d

(146).

An extensive infection with trichophyton rubrum would best be treated with


a. Tolnaftate
b. Griseofulvin
c. Nystatin
d. Miconazole
Solution. Ans-146: (b) Griseofulvin
Ref:Read the text below
Sol:
Trichophyton rubrum causes superficial fungal infection (dermatophytosis)

Mild to moderate superficial mycoses is treated with topical agents like miconazole, nystatin, tolnaftate etc. moderate to severe superficial mycoses (dermatophytosis) requires
an oral agent like griseofulvin (for best effect, in addition of topical agents.)

Correct Answer. b

(147).

A 50-year-old man presents with racing thoughts, insomnia, increasing impulsivity, and grandiosity for 1 month. A 1-month trial of
lithium with a blood level of 1.3 mEq/L fails to ameliorate his symptoms. Assuming no contraindications, which of the following is the
most appropriate next step in the management of this patient?
a. A trial of imipramine
b. A trial of valproate
c. An increase in the does of lithium
d. The addition of haloperidol
Solution. Ans-147: (b) A trial of valproate
Ref:Read the text below
Sol:
This patients condition is highly suggestive of bipolar disorder, which is not responsive to lithium treatment in approximately 25% of cases.

It would be unwise to increase the lithium dose in this patient because his level is already at the top of the therapeutic range.
The medication of choice in such circumstances is valproate, because some patients who do not respond to lithium will respond to valproate.
Imipramine would not be expected to control a manic episode.
Correct Answer. b

Copyright 2014 Delhi Academy of Medical Sciences, All Rights Reserved.

52/71

(148).

Intravenous amphotericin-B is the treatment of choice in


a. Dermal candidiasis
b. Cutaneous larva migrans
c. Disseminated forms of N.American blastomycosis
d. Onychomycosis
Solution. Ans-148: (c) Diseminated forms of N.American blastomycosis s
Ref:Read the text below
Sol:
I.V. amphotericin-B is the drug of choice in most systemic fungal infections e.g. blastomycosis, disseminated candidiasis, cryptococosis, histoplasmosis, cocidiodomycosis,
sporotrichosis, aspergillosis and mucormycosis.
The drugs of choice: in dermal candidiasis is nystatin/fluconazole, in cutaneous larva migrans (due to hook worms) is thiabendazole and in onychomycosis is griseofulvin (with
topical agents).

Correct Answer. c

(149).

A 48-year-old man with a history of alcoholism ingests a bottle of antifreeze and presents to the emergency department obtunded but
with intact vision. Which of the following is the most appropriate pharmacotherapy?
a. Amyl nitrite
b. Atropine
c. Ethanol
d. Glucagon
Solution. Ans-149: (c) Ethanol
Ref:Read the text below
Sol:
Ethanol is the appropriate treatment for patients who have ingested methanol or ethylene glycol.

This therapy is effective because ethanol competes for the same enzyme (alcohol dehydrogenase) that is required for methanol/ethylene glycol breakdown. Methanol can cause
blindness; ethylene glycol (a component of antifreeze) does not.
Therefore, the patient probably ingested ethylene glycol. Ethylene glycol is a CNS depressant; ingestion produces obtundation and unresponsiveness to painful stimuli.

Correct Answer. c

(150).

Which of the following has the shortest plasma half life


a. Propranolol
b. Esmolol
c. Timolol
d. Atenolol.
Solution. Ans-150: (b) Esmolol
Ref:Read the text below
Sol:
Esmolol is an ultrashort acting blocker.

It has half life < 10min.


Correct Answer. b

Copyright 2014 Delhi Academy of Medical Sciences, All Rights Reserved.

53/71

(151).

Selective beta-1 blocker is


a. Nadolol
b. Metoprolol
c. Pindolol
d. All of the above
Solution. Ans-151: (b) Metoprolol
Ref:Read the text below
Sol:
Metoprolol is blocker i.e. cardio-selective, like atenolol, acebutolol and bisoprolol.

Correct Answer. b

(152).

Which one of the following is not a selective beta-1 blocker


a. Esmolol
b. Celiprolol
c. Atenolol
d. Acebutalol
Solution. Ans-152: (b) Celiprolol
Ref:Read the text below
Sol:
Celiprolol is a 1 selective blocker but also has 2 agonistic action. The 2 agonist action makes it a useful drug in asthmatics (in contrast to other 2 blocker) and cause
additional vasodilatation.
The other blockers: Esmolol (ultrashort acting), atenolol and acebutalol are 1 selective blocker only, just as metoprolol and bisoprolol.

Correct Answer. b

(153).

Which of the following is a combined alpha and beta blocker?


a. Atenolol
b. Propranolol
c. Pindolol
d. Labetalol.
Solution. Ans-153: (d) Labetalol.
Ref:Read the text below
Sol:
Labetalol and carvedilol are combined alpha and beta blockers.

The other drugs listed are all beta blockers.


Correct Answer. d

Copyright 2014 Delhi Academy of Medical Sciences, All Rights Reserved.

54/71

(154).

Which is true of enalapril as compared to captopril


a. Lesser potent
b. Quicker onset
c. Longer acting
d. More adverse effects
Solution. Ans-154: (c) Longer acting
Ref:Read the text below
Sol:
Enalapril has to first convert into active enalaprilat, so have slower onset and therefore lesser first dose hypotension.

Correct Answer. c

(155).

Rebound hypertension is most commonly seen with


a. Thizide diuretics
b. Clonidine
c. Reserpine
d. Guanethidine.
Solution. Ans-155: (b) Clonidine
Ref:Read the text below
Sol:
Rebound increase in B.P. occurs if one or two doses of clonidine are missed.

This is because of sudden increase in plasma catecholamine levels due to:


Removal of sympathetic inhibition.
Super-sensitivity of peripheral adrenergic structures to catecholamines.
Correct Answer. b

(156).

Braycardia is seen with all of the following except


a. Methyldopa
b. Reserpine
c. Hydrallazine
d. Propranolol
Solution.
Ans-156: (c) Hydrallazine
Hydrallazine cause powerful arteriolar dilatation sharp vasodilatation reflex tachycardia.

Other agents mentioned have central sympatholytic effect, so decrease heart rate.
Correct Answer. c

Copyright 2014 Delhi Academy of Medical Sciences, All Rights Reserved.

55/71

(157).

Alered taste sensation is caused by


a. Verapamil
b. Nifedipine
c. Clonidine
d. Captopril.
Solution. Ans-157: (d) Captopril.
Ref:Read the text below
Sol:
Reversible loss or alteration of taste (=dysgeusia) has been noted with captopril in 0.5 to 3% patients.

Less incidence is seen with other ACE inhibitors.


Correct Answer. d

(158).

Which of the following acts directly on the blood vessels


a. Hydrallazine
b. Alpha methyldopa
c. Captopril
d. Propranolol.
Solution. Ans-158: (a) Hydrallazine
Ref:Read the text below
Sol:
Hydrallazine acts directly on vascular smooth muscles NO or cCMP or Ca2+---- vascular smooth muscular relaxation vasodilatation (mainly arteriolar dilatation).

Correct Answer. a

(159).

The dose of theophylline should be increased in case of


a. Phenytoin use
b. Cor-pulmonale
c. Hepatic impairment
d. Allopurinol use
Solution. Ans-159: (a) Phenytoin use
Ref:Read the text below
Sol:
Dose of theophyline should be increased with drugs which induce its metabolism e.g Phenytoin

Correct Answer. a

Copyright 2014 Delhi Academy of Medical Sciences, All Rights Reserved.

56/71

(160).

The longest acting bronchodilator is


a. Salbutamol
b. Bitolterol
c. Terbutaline
d. Ritodrine.
Solution. Ans-160: (b) Bitolterol
Ref:Read the text below
Sol:
Bitolterol produces longer period of bronchodilatation due to formation of an active metabolite colterol.

However, the disagreeable taste of bitolterol has discouraged widespread use.


Salmeterol is now popular, due to its long 12 hour duration of action.
It is especially preferred in the prophylaxis of nocturnal asthma.
Correct Answer. b

(161).

In chronic obstructive airway disease the following are used except


a. Bromhexine
b. Aminophylline
c. Steam inhalation
d. 100% oxygen
Solution. Ans-161: (d) 100% oxygen.
Ref:Read the text below
Sol:
100% oxygen decreases clearance of mucus from the respiratory tract and also has other deleterious effects.

Correct Answer. d

(162).

The drug used in bronchial asthma is


a. Cimetidine
b. Methyl xanthine
c. Propranolol
d. Dihydroxy quinoline.
Solution. Ans-162: (b) Methyl xanthine
Ref:Read the text below
Sol:
Methyl xanthines such as aminophylline are useful bronchodilators.

Correct Answer. b

Copyright 2014 Delhi Academy of Medical Sciences, All Rights Reserved.

57/71

(163).

Budesonide is
a. A newer antiepileptic
b. A newer antihypertensive
c. A newer inhalational steroid
d. A newer bronchodilator.
Solution. Ans-163: (c) A newer inhalational steroid
Ref:Read the text below
Sol:
Budesonide is a glucocorticoid steroid for the treatment of asthma, non infectious rhinitis(including hay fever and other allergies), and for treatment and prevention of nasal
polyposis.
Additionally, it is used for Crohn's disease (inflammatory bowel disease).

Correct Answer. c

(164).

In case of respiratory distress in child, which drug is not to be given?


a. Terbutaline
b. Aminophylline
c. Morphine
d. Co-trimoxazole.
Solution. Ans-164: (c) Morphine
Ref:Read the text below
Sol:
Morphine is a respiratory depressant, hence contraindicated.

Terbutaline (beta-2 agonist) and aminophylline are bronchodilators, so may benefit the respiratory distress.
Co-trimoxazole, an antibacterial, is useful in lung infections causing respiratory distress.
Correct Answer. c

(165).

A new antibiotic is being tested. The following pharmacokinetic parameters have been determined.
Clearance = 100 ml/min
Volume of distribution (Vd) = 50 L
Half-life = 3 hours
Assuming that the drug is being administered intravenously, what loading dose (LD) should be given to a patient to quickly obtain a
plasma concentration of 10 mg/L?
a. 5mg
b. 25mg
c. 100mg
d. 500mg
Solution. Ans-165: (d) 500mg
Ref:Read the text below
Sol:
The object of a loading dose (LD) is to load up the volume of distribution in order to quickly achieve the desired plasma concentration.

You can calculate it by the following equation.


LD = Vd X Cp
(Vd volume of distribution Cp = Plasma concentration)
In this case, LD = (50L) x (10mg/L) = 500mg

Correct Answer. d

Copyright 2014 Delhi Academy of Medical Sciences, All Rights Reserved.

58/71

(166).

A 52-year-old man with peptic ulcer disease has been on drug therapy for 3 months and has noticed changes in his bowel habits,
increasing headaches, dizziness, skin rashes, loss of libido, and gynecomastia. Which of the following drugs is most likely responsible for
these side effects?
a. Cimetidine
b. Famotidine
c. Metronidazole
d. Omeprazole
Solution. Ans-166: (a) Cimetidine
Ref:Read the text below
Sol:
Cimetidine, an H2 receptor antagonist, can produce all the side effects exhibited when taken in high does over a long period of time. In addition, cimetidine can alter the
hepatic metabolism of several drugs by inhibiting CYP450 enzyme.
Famotidine is also an H2 receptor antagonist, but it does not have the side effects of cimetidine.

Correct Answer. a

(167).

In a study of a new drug, the agent was administered to anesthetized animals while blood pressure, heart rate, and salivation were
recorded. The results of a typical experiment are shown in Fig. What is the best characterization of this new agent?

a. Alpha-adrenoceptor agonist
b. Alpha-adrenoceptor antagonist
c. Cholinesterase inhibitor
d. Direct-acting muscarinic agonist
Solution. Ans-167: (d) Direct-acting muscarinic agonist
Ref: Read the text below
Sol :
Salivary glands contain muscarinic receptors, primarily of the M3 subtype, that receive
parasympathetic innervations.
Direct-acting agonists such as bethanechol and indirect agents such as neostigmine mimic parasympathetic nerve stimulation.

Blood vessel endothelial cells contain M3 receptors that are not innervated, but respond to circulating directacting muscarinic agonists.
When these endothelial receptors are activated, nitric oxide synthesis is stimulated and smooth muscle relaxation occurs promptly with vasodilation and a drop in blood
pressure. Because no nerve endings are present, indirect-acting cholinomimetics such as cholinesterase inhibitors do not have this vasodilating effect.

Correct Answer. d

Copyright 2014 Delhi Academy of Medical Sciences, All Rights Reserved.

59/71

(168).

Beta blocker with peripheral vasodilator action is


a. Carvedilol
b. Propranolol
c. Atenolol
d. Acebutolol
Solution. Ans-168 : (a) Carvedilol
Ref: Goodman & Gilman -285, 286
Sol :
Third generation -blockers possess additional vasodilator activity apart from their -blocking action.

Carvedilol and labetalol block receptors also and cause vasodilation.


Correct Answer. a

(169).

Which is not an endogenous catecholamine?


a. Dopamine
b. Dobutamine
c. Adrenaline
d. Noradrenaline
Solution. Ans-169: (b) Dobutamine
Ref: Read the text below
Sol :
Catecholamines are the compounds containing catechol nucleus and side chain containing amine

Dopamine, adrenaline and nonadrenaline are endogenous catecholamines.


Isoprenaline, dipivefrine, dobutamine, ibopamine, dopexamine and fenoldopam are the exogenous (synthetics) catecholamines.
Correct Answer. b

(170).

Propanolol can be used in all of the following conditions except


a. Thyrotoxicosis
b. Variant angina
c. Migraine
d. Hypertension
Solution.
Ans-170: (b) Variant angina
Ref: KDTs - 129
Sol :
Propanolol worsens variant angina, due to unopposed receptor mediated coronary constriction.s

Correct Answer. b

Copyright 2014 Delhi Academy of Medical Sciences, All Rights Reserved.

60/71

(171).

Half life of Dobutamine is


a. 120 sec.
b. 200 sec
c. 20 sec.
d. 20 min.
Solution. Ans-171: (a) 120 sec.
Ref: Read the text below
Sol :
Dobutamine is an inotropic drug which has onset of action =1-2 min.

Peak action = 1-10 min.


Duration of action = <10 min.
Half life = 2 min.
Correct Answer. a

(172).

Which of these statements is true in context of dobutamine


a. It is a strong vasoconstrictor
b. It significantly increases heart rate
c. It is a potent broncholdilator
d. It reduces afterload
Solution.thAns-172: (a) It is a strong vasoconstrictor

Ref: KDTs 5 - 112


Sol :
Dobutamine acts on both alpha and beta adrenergic receptors.

The only prominent action of clinical significance is increase in force of cardiac contraction and output without significant change in heart rate, peripheral resistance (no
vasodilation and constriction) and BP.
It has relatively selective 1 action so no effect on bronchi.

Dobutamine is a derivative of dopamine but no D1 or D2 action.


Dobutamine consist of two isomers.
(+) isomer is potent 1agonist & 1 antagonist.
() isomer is a potent 1 agonist capable of causing significant vasoconstriction when given alone.
Correct Answer. a

(173).

Atropine causes all of the following actions except


a. Increases bowel sound
b. Decreases bowel sound
c. Causes bronchodilatation
d. Causes mydriasis
Solution. Ans-173: (a) Increases bowel sound
Ref: KDTs - 94
Sol :
Atropine is an anticholinergic drug. It blocks all type of muscarinic receptors.

Topical instillation causes mydriasis, abolition of light reflex and cycloplegia. IOT tends to rise.
All visceral smooth muscles are relaxed. Tone and amplitude of contraction of stomach and intestine are reduced causing constipation.
Atropine causes bronchodilation and reduction of airway resistance in COPD and asthma patient.
Atropine decreases all body secretions.
Atropine causes tachycardia due to M2 blockade.
Correct Answer. a

Copyright 2014 Delhi Academy of Medical Sciences, All Rights Reserved.

61/71

(174).

Which of the following is the ultrashort acting cardioselective adrenergic blocker.


a. Bisoprolol
b. Timolol
c. Sotalol
d. Esmolol
Solution. Ans-174: (d) Esmolol
Ref: KDTs - 141
Sol :
Esmolol (trade name Brevibloc) is a cardioselective beta1 receptor blocker with rapid onset, a very short duration of action, and no significant intrinsic sympathomimetic or
membrane stabilising activity at therapeutic dosages.
It is a class II antiarrhythmic.

Esmolol decreases the force and rate of heart contractions by blocking beta-adrenergic receptors of the sympathetic nervous system, which are found in the heart and other
organs of the body. Esmolol prevents the action of two naturally occurring substances: epinephrine and norepinephrine.
It is the shortest acting blocker.

Correct Answer. d

(175).

All of the following are features of metoprolol in comparison to propranolol except


a. It is ineffective in suppressing muscle tremor
b. It is safer in diabetics
c. It is less likely to cause bradycardia
d. It is less likely to worsen Raynauds disease
Solution. Ans-175: (c) It is less likely to cause bradycardia
Ref: KDTs -141
Sol :
Metoprolol is a cardio-selective locker and thus is safer than non-selective blockers (like propranolol) in diabeties, asthmatics and the patients of peripheral vascular disease
(e.g. Raynauds disease).
Due to lack of 2 blocking action, it cannot suppress muscle tremors. Potential to cause bradycardia is similar to non-selective blockers.

Drugs possessing ISA (like pindolol) are less likely to cause bradycardia.
Correct Answer. c

(176).

The neurotransmitter agent that is normally released in the SA node of the heart in response to increased blood pressure is
a. Acetylcholine
b. Dopamine
c. Adrenaline
d. Noradrenaline
Solution. Ans-176: (a) Acetylcholine
Ref: KDTs - 108
Sol :
Whenever blood pressure increases there is reflex stimulation of baroreceptors.

These release Ach and depresses the heart.


Correct Answer. a

Copyright 2014 Delhi Academy of Medical Sciences, All Rights Reserved.

62/71

(177).

A highway truck driver has profuse rhinorrhoea and sneezing. Which amongst the following drugs would you prescribe him?
a. Pheniramine
b. Promethazine
c. Dimenhydrinate
d. Cetrizine
Solution. Ans-177: (d) Cetrizine
Ref: KDTs - 157
Sol :
It is a non-sedating second generation antihistaminic agent and is preferred for the patients requiring constant attention.

It does not cause sedation or impairment of psychomotor function.


Other drugs listed in the question are first generation antihistaminics.
Correct Answer. d

(178).

Which NSAID undergoes enterohepatic circulation?


a. Phenylbutazone
b. Aspirin
c. Ibuprofen
d. Piroxicam
Solution. Ans-178: (d) Piroxicam
Ref: KDT- 178
Sol :
Piroxicam is a non-steroidal anti-inflammatory drug of the oxicam class used to relieve the symptoms of rheumatoid and osteoarthritis, primary dysmenorrhoea, postoperative
pain; and act as an analgesic, especially where there is an inflammatory component.
Piroxicam is a long acting analgesic due to enterohepatic circulation. Single daily dose is usually sufficient.

Piroxicam use can result in gastrointestinal toxicity, tinnitus, dizziness, headache, rash, and pruritus. The most severe adverse reactions are peptic ulceration, gastrointestinal
bleeding, and severe skin reactions including StevensJohnson syndrome and toxic epidermal necrolysis. Approximately 30% of all patients receiving daily doses of 20 mg of
piroxicam experience side effects

Correct Answer. d

(179).

The H3 receptor agonist exhibits all of the following actions except


a. Inhibition of H1 receptor induced wakefulness
b. Increase in H1 mediated gastrin secretion
c. Inhibition of H1 mediated bronchoconstriction
d. Negative chronotropic effect on atria
Solution. Ans-179: (b) Increase in H1 mediated gastrin secretion
Ref: Read the text below
Sol :
H3receptors act as pre-synaptic receptors and decrease the release of histamine and other neurotransmitters.

These antagonize H1 mediated wakefulness and broncho constriction.


These also decrease gastrin release and produce negative chronotropic effect.
Correct Answer. b

Copyright 2014 Delhi Academy of Medical Sciences, All Rights Reserved.

63/71

(180).

Which of the following will be optium treatment in a patient of chronic hepatitis B whose serum AST levels are raised :a. Lamivudine
b. Lamivudine + interferons
c. Immunoglobulins
d. No treatment required
Solution. Ans-180: (a) Lamivudine
Ref: Read the text below
Sol :

Correct Answer. a

(181).

Choose the correct pair regarding drug mechanism:a. Cycloserine Cause leakage from cell membrane
b. Rifampin Interfere with DNA function.
c. Colistin- Inhibits cell wall synthesis
d. Zidovudine- Interfere with RNA synthesis.
Solution. Ans-181: (b) Rifampin Interfere with DNA function.
Ref: Read the text below
Sol :
Inhibits cell wall synthesis: Penicillins,Cephalosporins, Cycloserine, Vancomycin, bacitracin.
B. Cause leakage from cell membranes:
PolypeptidesPolymyxins, Colistin, Bacitracin.
PolyenesAmphotericin B, Nystatin, Hamycin.
C. Inhibit protein synthesis: Tetracyclines, Chloramphenicol, Erythromycin, Clindamycin, Linezolid.
D. Causes misreading of m-RNA code and affect permeability: AminoglycosidesStreptomycin, Gentamicin etc.
E. Inhibit DNA gyrase: Fluoroquinolones-Ciprofloxacin.
F. Interfere with DNA function: Rifampin, Metronidazole.
G. Interfere with DNA synthesis: Idoxuridine, Acyclovir, Zidovudine.
H. Interfere with intermediary metabolism: Sulfonamides, Sulfones, PAS, Trimethoprim, Pyrimethamine,
Ethambutol.

Correct Answer. b

Copyright 2014 Delhi Academy of Medical Sciences, All Rights Reserved.

64/71

(182).

Least nephrotoxic drug is:


a. Netilmicin
b. Sisomicin
c. Tobramycin
d. Amikacin
Solution. Ans-182: (c) Tobramycin
Ref: Read the text below
Sol :

Correct Answer. c

(183).

Which of the following is not a non-sedative anti-allergic drug?


a. Cetirizine
b. Astemizole
c. Terfenadine
d. Triprolidine
Solution. Ans-183: (d) Triprolidine
Ref: KDTs - 179
Sol :
Terfenadine, astemizole and cetirizine are non- sedating second generation anti-allergic drugs.

Triprolidine is a mild sedative anti-histamnic.


Correct Answer. d

(184).

Which of the following drugs is useful in acute attack of gout?


a. Furosemide
b. Sulfinpyrazone
c. Allopurinol
d. Piroxicam
Solution. Ans-184: (d) Piroxican
Ref: KDTs - 194
Sol :
NSAIDs and colchincine are highly effective in acute attack of gout whereas allopurinol and sulfirpyrazone are used for chronic gout.

Furosemide causes hyperuricemia and should be a avoided in patients with gout.


Correct Answer. d

Copyright 2014 Delhi Academy of Medical Sciences, All Rights Reserved.

65/71

(185).

Which of the following statements most aptly characterizes ibuprofen vis--vis other NSAIDs
a. It is less effective in migraine as compared to other NSAIDs
b. It is more effective in dysmenorrheal than other NSAIDs
c. It is less effective than other NSAIDS in the treatment of food hypersensitivity
d. Like other NSAIDs it can cause aseptic meningitis
Solution. Ans-185: (d) Like other NSAIDs it can cause aseptic meningitis
Ref: KDTs - 176
Sol :
Ibuprofen
Ibuprosfen is used as a simple analgesic and antipyretic in the same way as low doses of aspirin.

It is particularly effective in dysmenorrheal in which action is clearly due to PG synthesis inhibition.


NSAIDs like ibuprofen, naproxen, diclofenac, mephenamic acid and indomethacin etc. are effective in mild migraine.
Aseptic meningitis may follow systemic drug administration. Drug induced meningitis has been observed following administration of NSAIDs (esp. ibuprofen) and H2 receptor
antagonists.

Correct Answer. d

(186).

Which of the following drugs inhibit platelet cyclo-oxygenase reversibly?


a. Alprostadil
b. Aspirin
c. Ibuprofen
d. Prednisolone
Solution. Ans-186: (c) Ibuprofen
Ref: KDTs - 185
Sol :
Alprostadil (PGE1) and prednisolone to not inhibit COX enzyme whereas aspirin is an irreversible inhibitor of this enzyme.

Ibuprofen is used primarily for fever, pain, dysmenorrhea and inflammatory diseases such as rheumatoid arthritis.
It is also used for pericarditis and patent ductus arteriosus
Correct Answer. c

(187).

Which of the following is a component of slow reacting substance of anaphylaxis (SRS-A)?


a. LTC4
b. Misoprostol
c. Prostacyclin
d. None.
Solution.thAns-187: (a) LTC4

Ref: KDTs 6 Ed. Pg. 176


Sol :
LTD4, D4 and E4 are known as slow reacting substance of anaphylaxis as these can cause bronchoconstriction.

Correct Answer. a

Copyright 2014 Delhi Academy of Medical Sciences, All Rights Reserved.

66/71

(188).

A drug X is useful in the treatment of rheumatoid arthritis. It is available only in parenteral formulation and its mechanism of action is
antagonism of tumor necrosis factor. Which of the following can be X?
a. Cyclosporine
b. Penicillamine
c. Phenylbutazone
d. Etanercept
Solution. Ans-188: (d) Etanercept
Ref: KDTs - 205
Sol :
Infliximab and etanercept are TNF- antagonists useful for the treatment of rheumatoid arthritis.

These are administered by parenteral route.


TNF - antagonists can cause reactivation of latent tuberculosis.
Etanercept is a biopharmaceutical that treats autoimmune diseases by interfering with tumor necrosis factor (TNF; a soluble inflammatory cytokine) by acting as a TNF
inhibitor. It has U.S. F.D.A. approval to treat rheumatoid, juvenile rheumatoid and psoriatic arthritis, plaque psoriasis and ankylosing spondylitis. TNF-alpha is the "master
regulator" of the inflammatory (immune) response in many organ systems. Autoimmune diseases are caused by an overactive immune response. Etanercept has the potential to
treat these diseases by inhibiting TNF-alpha

Correct Answer. d

(189).

Which of the following is not given alone in a patient of pheochromocytoma?


a. Atenolol
b. Prazosin
c. Nitroprusside
d. Metyrosine
Solution. Ans-189: (a) Atenolol
Ref: KDTs - 142
Sol :
Beta blockers should never be given alone (or before blockers) in pheochromocytoma.

There is excess of catecholamines in the circulation in this condition. They increase BP by acting on receptors. Beta blockers will result in further increase in blood pressure
by antagonizing 2 mediated vasodilatation.
Thus, the patient may end in hypertensive crisis. To avoid this complication, -blockers should be given before blockers or combined and - blockers should be given.

Correct Answer. a

Copyright 2014 Delhi Academy of Medical Sciences, All Rights Reserved.

67/71

(190).

Maximum incidence of impotence is seen with the following anti-hypertensive agent


a. CCBs
b. Beta blockers
c. Renal failure
d. ACE inhibitors
Solution. Ans-190: (b) Beta blockers
Ref: Read the text below
Sol :
Vasodilation is required for erection of penis. Non-selective blockers inhibit the 2 mediated vasodilation and may result in erectile dysfunction and impotence.
Drugs implicated in causation of erectile dysfunction are :
-blockers

Clonidine
-methyldopa
Thiazides
Antipsychotics e.g. phenothiazines
MAO inhibitors
Benzodiazepines
Antiandrogens
Estrogens
Alcohol
Phosphodiesterase inhbibitors (sildenafil, tadalafil and vardenafil), 2 blockers (Yohimbine), Non-selective blockers (phentolamine), dopamine analog (apomorphine), PGE1
analog (alprostadil) and trazodone are used for the treatment of erectile dysfunction.

Correct Answer. b

(191).

Which of the following is not given alone in a patient with pheochromocytoma:


a. Atenolol
b. Prazosin
c. Nitroprusside
d. Metyrosine
Solution. Ans-191: (a) Atenolol
Ref:Read the text below
Sol:
Pheochromocytoma is a tumour releasing catecholamines causing very high blood pressure. Please note that receptors mediate vasodilatation while receptors cause
vasoconstriction.
Therefore, in this condition, if we give only beta blocker (e.g. Atenolol) then unmasked mediated vasoconstriction would very much exacerbate the hypertension.

Drugs prescribed in pheochromocytoma are :


-Blockers e.g. Prazosin, Phenoxybenzamine, Phentolamine;
Direct vasodilators e.g. Sod. Nitroprusside;
Catecholamine synthesis inhibitor e.g. Metyrosine.
Correct Answer. a

(192).

A short acting narcotic is


a. Morphine
b. Pethidine
c. Alfentanil
d. Fentanyl
Solution. Ans-192: (c) Alfentanil
Ref:Read the text below
Sol:
Alfentanil is a shorter acting (dur <1hr) analogue of fentanyl (duration 1-2 hr.) Duration of action of morphine is 4-6 hrs, while that of pethidine is 3-4 hrs.

Correct Answer. c

Copyright 2014 Delhi Academy of Medical Sciences, All Rights Reserved.

68/71

(193).

Which of the following alkaloids is not present in opium


a. Cyclazocine
b. Codeine
c. Thebaine
d. Papaverine
Solution. Ans-193: (a) Cyclazocine
Ref:Read the text below
Sol:
Cyclazocine is not present in opium.

Opium contains phenanthrene derivatives (morphine, codeine and thebaine) and benzoisoquinoline derivatives (noscapine and papaverine).
Correct Answer. a

(194).

Heroin differ from morphine in all respects, except


a. Causes more euphoria
b. More constipation
c. Slowly metabolized to morphine
d. Synthetic congener of morphine
Solution. Ans-194: (b) More constipation
Ref:Read the text below
Sol:
Heroin, a semisynthetic congener of morphine, is more lipid soluble.

So, it enters brain more rapidly and cause more enphoria, specially on I.V. injection (Kick).
However, peripheral effects like constipation, hypotension are similar or less. Heroin (diacetyl morphine) is first metabolized to morphine and then glucuronidation occurs.
Correct Answer. b

(195).

Which of the following antirheumatic agents has the least incidence of gastrointestinal upset and bleeding
a. Aspirin
b. Dexamethasone
c. Aurothiomalate
d. Indomethacin
Solution. Ans-195: (c) Aurothiomalate
Ref:Read the text below
Sol:
Gold salts like aurothiomalate do not cause acid peptic diseases while both steroidal (e.g. dexamethasone) and non steroidal anti-inflammatory drugs (e.g. aspirin and
indomethacin) are known to cause the same.

Correct Answer. c

Copyright 2014 Delhi Academy of Medical Sciences, All Rights Reserved.

69/71

(196).

A patient on diuretics, develops gynecomastia and hirsutism. The likely drug is


a. Chlorthiazide
b. Frusemide
c. Spironolactone
d. Acetazolamide
Solution. Ans-196: (c) Spironolactone
Ref:Read the text below
Sol:
Spironolactone an aldosterone antagonist, also interfere with hormonal functions.

It may cause gynecomastia, hirsutism, impotence and menstrual irregularities.


Correct Answer. c

(197).

A patient with a history of glaucoma, epilepsy, and edema would be a candidate for treatment with which of the following diuretics?
a. Ethacrynic acid
b. Chlorothiazide
c. Furosemide
d. Acetazolamide
Solution. Ans-197: (d) Acetazolamide
Ref:Read the text below
Sol:
The carbonic anhydrase inhibitor, acetazolamide, although a weak diuretic, is useful in the treatment of glaucoma and petit mal epilepsy.

The high ceiling diuretics, namely ethacrynic acid and furosemide and the thiazide diuretic, chlorothiazide are not effective agents in the treatment of glaucoma or epilepsy,
although they are more effective as diuretics.

Correct Answer. d

(198).

All of the following are true regarding diuretics except:


a. Spirionolactone is associated with hypokalemia
b. Acetazolamide inhibits carbonic anhydrase
c. Furosemide causes hypokalemia
d. Thizaides cause hypercalcemia
Solution. Ans-198: (a) Spirionolactone is associated with hypokalemia
Ref:Read the text below
Sol:
+
Spirinolactone is a potassium sparing diuretic, so it retains the body K and causes hyperkalemia.

Acetazolamide
is a weak diuretic which acts by inhibiting carbonic anhydrase enzyme in the proximal tubular cells. Furosemide is a strong loop diuretic which depletes body
2+
2+
2+
Na+, K+ Ca & Mg
Thiazides in contrast to loop diuretics, reduces Ca
excretion and cause hypercalcemia.
Therefore, Thiazides are used in patients of hyper calciuria who tend to develop renal calcium

Correct Answer. a

Copyright 2014 Delhi Academy of Medical Sciences, All Rights Reserved.

70/71

(199).

Bronchodilator effect without significant cardiac effect is the characteristic feature of


a. Isoprenaline
b. Isoxsuprine
c. Propranolol
d. Salbutamol
Solution. Ans-199: (d) Salbutamol
Ref:Read the text below
Sol:
Salbutamol is a specific bronchodilator as it is a 2 selective agonist.

Correct Answer. d

(200).

Which of the following drugs is not used in the treatment of acute attack of bronchial asthma
a. Nifedipine
b. Orciprenaline
c. Salbutamol
d. Theophylline
Solution. Ans-200: (a) Nifedipine
Ref:Read the text below
Sol:
Nifedipine a calcium channel blocker, is not useful in the acute attack of bronchial asthma

Correct Answer. a

Test Answer
1.(b)

2.(d)

3.(a)

4.(c)

5.(c)

6.(b)

7.(c)

8.(d)

9.(b)

10.(b)

11.(d)

12.(a)

13.(a)

14.(b)

15.(c)

16.(c)

17.(b)

18.(b)

19.(a)

20.(b)

21.(c)

22.(a)

23.(a)

24.(c)

25.(c)

26.(a)

27.(a)

28.(b)

29.(a)

30.(b)

31.(b)

32.(a)

33.(d)

34.(d)

35.(b)

36.(b)

37.(a)

38.(c)

39.(d)

40.(b)

41.(b)

42.(b)

43.(a)

44.(a)

45.(a)

46.(c)

47.(b)

48.(d)

49.(d)

50.(a)

51.(c)

52.(a)

53.(c)

54.(c)

55.(a)

56.(d)

57.(a)

58.(c)

59.(d)

60.(a)

61.(a)

62.(a)

63.(a)

64.(d)

65.(c)

66.(c)

67.(a)

68.(a)

69.(d)

70.(b)

71.(a)

72.(b)

73.(a)

74.(c)

75.(d)

76.(b)

77.(c)

78.(c)

79.(c)

80.(b)

81.(c)

82.(a)

83.(d)

84.(b)

85.(c)

86.(b)

87.(a)

88.(a)

89.(a)

90.(a)

91.(c)

92.(d)

93.(b)

94.(a)

95.(a)

96.(d)

97.(c)

98.(d)

99.(b)

100.(b)

101.(a)

102.(b)

103.(a)

104.(a)

105.(c)

106.(a)

107.(a)

108.(b)

109.(c)

110.(d)

111.(d)

112.(d)

113.(b)

114.(b)

115.(c)

116.(a)

117.(b)

118.(a)

119.(b)

120.(c)

121.(d)

122.(d)

123.(c)

124.(a)

125.(c)

126.(c)

127.(c)

128.(a)

129.(c)

130.(c)

131.(c)

132.(c)

133.(b)

134.(c)

135.(b)

136.(d)

137.(a)

138.(b)

139.(b)

140.(a)

141.(a)

142.(b)

143.(d)

144.(d)

145.(d)

146.(b)

147.(b)

148.(c)

149.(c)

150.(b)

151.(b)

152.(b)

153.(d)

154.(c)

155.(b)

156.(c)

157.(d)

158.(a)

159.(a)

160.(b)

161.(d)

162.(b)

163.(c)

164.(c)

165.(d)

166.(a)

167.(d)

168.(a)

169.(b)

170.(b)

171.(a)

172.(a)

173.(a)

174.(d)

175.(c)

176.(a)

177.(d)

178.(d)

179.(b)

180.(a)

181.(b)

182.(c)

183.(d)

184.(d)

185.(d)

186.(c)

187.(a)

188.(d)

189.(a)

190.(b)

191.(a)

192.(c)

193.(a)

194.(b)

195.(c)

196.(c)

197.(d)

198.(a)

199.(d)

200.(a)

Copyright 2014 Delhi Academy of Medical Sciences, All Rights Reserved.

71/71

Vous aimerez peut-être aussi